From owner-texhax-digest@nottingham.ac.uk Mon Jan 8 15:02:41 1996 Flags: 000000000001 Received: from jess.ccc.nottingham.ac.uk (jess.ccc.nottingham.ac.uk [128.243.40.193]) by csc-sun.math.utah.edu (8.7.1/8.7.1) with ESMTP id PAA16705 for ; Mon, 8 Jan 1996 15:00:38 -0700 (MST) Message-Id: <199601082200.PAA16705@csc-sun.math.utah.edu> Received: from nottingham.ac.uk by jess.ccc.nottingham.ac.uk id <11536-0@jess.ccc.nottingham.ac.uk>; Mon, 8 Jan 1996 20:43:04 +0000 From: Majordomo list server To: texhax-digest@nottingham.ac.uk Subject: TeXhax Digest V96 #1 Reply-To: TeXhax@tex.ac.uk Errors-To: owner-texhax-digest@nottingham.ac.uk Precedence: bulk Date: Mon, 8 Jan 1996 20:43:04 +0000 Sender: owner-texhax-digest@nottingham.ac.uk TeXhax Digest Monday, 8 January 1996 Volume 96 : Number 001 (incorporating UKTeX Digest) Today's Topics: Re: TeXhax Digest V95 #18 Re: TeXhax Digest V95 #18 some questions from a newcomer more questions! LaTeX2e modes.mf 3.0 available ---------------------------------------------------------------------- From: Sebastian Rahtz Date: Mon, 18 Dec 1995 09:16:38 GMT Subject: Re: TeXhax Digest V95 #18 > From: Andre HECK > Date: Tue, 05 Dec 95 17:25:31 +0100 > Subject: Chemical TeX/LaTeX > > My daughter heard that specific chemistry-oriented TeX/LaTeX packages > were available, without more details unfortunately. My search in the > ctan archives were unsuccessful. > Anyone knows more about this matter? If so, is there any shareware > available? there are several; look for chemtex, and xymtex; the latter, from Japan, is described in an article in the next issue of TUGboat. Another package, described by J Hagen at this years EuroTeX, is published in the EuroTeX 95 proceedings, an article which will be reprinted in TUGboat early next year Sebastian Rahtz Secretary, TUG ------------------------------ From: lamport@pa.dec.com Date: Sun, 17 Dec 95 13:58:13 -0800 Subject: Re: TeXhax Digest V95 #18 J. Greg Davidson, who hates being misquoted, writes I find that most people I show TeX to dislike it on sight and decline my offer to help them learn to use it. At the same time, they're interested in SGML and planning to convert existing documents to and write new documents in that form. This is the source of our big opportunity. My biggest gripe about TeX is that I find writing and understanding TeX macros to be difficult... As people move towards keeping documents in SGML form, they will be looking for a good text formatter to use in conjunction with their SGML documents. Many people think that SGML offers a solution to the problems of document preparation. They're wrong. SGML is just a statement of the problem. The goals of SGML--a markup language that separates logical structure from formatting directions--are precisely the goals of LaTeX. However, LaTeX is a program to produce real output--not just a syntax for wishing what the document should look like. It is this difference that is the primary reason why LaTeX input is a lot uglier than SGML; the difference between TeX syntax and SGML syntax has little to do with it. SGML syntax has some nice features. However, if you look at the HTML3.0 syntax for describing equations, you will discover that no-one in his right mind would want to type any significant amount of mathematics in SGML. Should HTML ever become expressive enough for mathematical documents (at the moment, the only math symbols HTML3.0 provides are \sqrt, \lnot, and \pm), one would have to create those documents either with a WYSIWYG editor or by translating from a more TeX-like syntax. We've learned a lot in the last dozen years, and we know how to do some things better than Don did in TeX. But it's by no means clear that we know how to create a system with the power of TeX that would make adding new features very much easier. Think of all the work that's been done to give unsophisticated users the power of a programming language. After 30 years, there still doesn't seem to be anything better than Basic. Leslie Lamport ------------------------------ From: gabriele@dtovf1.roma2.infn.it (gabriele migliorini) Date: Wed, 27 Dec 95 15:27:26 +0100 Subject: some questions from a newcomer Dear readers, i just began working with LaTeX, and I am waiting for a friend of mine to take me back from England a copy of the Companion (here in Italy, English and american books cost twice their price, for you get a very unfair exchange rate from bookshops) I'd like to get some little help before getting the book, even if I fear my questions could seem too much stupid for the expert ones. Two kind of problems: Text: how do I do to write a paragraph (or some paragraphs) and keep all of them on the same page? I remember that (a long time ago) in word perfect there was a function to put in a couple of markers telling to keep all the text in beetween on the same page. By now, I tried to resolve my problem by putting the paragraph in a minipage, but this cannot function with in paragraph stuff. Graphics: I poked around to understand how to include graphics in my documents, and I found many suggestions. Now, I've seen that (to me) the best format are those whose commands ger embedded right into the dvi file. I've seen that pstricks and eepic do thins kind of job, but both are written for LaTeX209; I cannot let pst function, but eepic seems great, with both XFIG and gnuplot. Now, my question: is there anybody who knows if there are incompatibilities w/epic-eepic other than the ones I didn't see? And what about PSTricks? Now, I have to thank everybody, and tell an happy new year to all the readers, those that will help me, and those who will be bored by my English an my questions. I apologize!. Sincerely, gabriele gabriele@dtovf1.roma2.infn.it ------------------------------ From: gabriele@dtovf1.roma2.infn.it (gabriele migliorini) Date: Wed, 27 Dec 95 16:17:26 +0100 Subject: more questions! I remembered just now that I need one more information about LaTeX: I'm writing about rotary motions, and I enocounter many rotations vectors; the standard practice is to use the normal greek letters you'd use for an angle, and tell the vector character typing the name in bold face. I can do that with standard {\bf a}, but the command {\bf \omega} doesn't work. How can I fix this behavior? Now, I'm to ask if there is someone willing to tell me the reason why the \left and \right verbs before opening and closing brackets and braces cannot be implicitly told by LaTeX: if i'm writing a fraction like (\frac {abc}{def}), I get small brackets; don't you think the standard behavior should be to prepone an mplicit \left or \right to any opening and closing bracket? (I see that |a|b|c| could break this semantics, but I think you could make the bars use the explicit notations and the other delimitations the implicit one) I thank you, sincerely. gabriele ------------------------------ From: Andrew Jones $STAFF Date: Wed, 20 Dec 1995 16:29:11 +0000 Subject: LaTeX2e To whom it may concern, I have a few problems which need solving, and do not know where to go for advice: I currently possess just LaTeX, but wiped it off my system, when trying to make room for something else. Stupidly, I did not back it up and recall that I had some problems trying to install it last time, having to change various lines in various files, both in emtex, and in my Config.sys and autoexec.bat files. As you may gather, I am pretty well a beginner with TeX. Does anyone know of anyone who would be able guide me through the reinstallation? Secondly, when I originally installed it, I had it set up to print on a Star LC10 9-pin dot matrix. I now possess a Canon BJ-10ex. I managed to get various printer drivers, none written speciically for this model, but none of them seem to work. Any suggestions? Finally, I was given your address from a visiting lecturer who uses LaTeX enormously. It rekindled my interest in Latex, and I would dearly love to get it properly re-installed once again. However, I notice that LaTeX2e is mentioned a great deal. What are the advantages, if any, of switching to this? I am currently using a 386sx-20 with a 70 MB HDD, but am hoping to upgrade to a 486DX2-66 with a 400MB HDD and CD-ROM in the near future. I would gratefully appreciate any information that you can give me. Yours sincerely Andrew Jones at Trent College the near future ------------------------------ From: "K. Berry" Date: Mon, 8 Jan 1996 14:55:22 -0500 Subject: modes.mf 3.0 available I have released version 3.0 of modes.mf. You can get it by anonymous ftp from ftp.cs.umb.edu:pub/tex/modes.mf and shortly from ftp.cdrom.com:pub/tex/modes.mf and the ctan sites in /tex-archive/fonts/modes/modes-3.0.mf. finger ctan@ftp.shsu.edu for a list of all the CTAN sites and mirrors. The mailing list tex-archive@math.utah.edu receives most TeX-related announcements; email tex-archive-request@math.utah.edu to join. You can also get it by email from if you cannot ftp: email fileserv@shsu.edu with a body of `sendme modes'. News: - - most importantly, a new and much cleaner way of doing write/white changes, compatible with the DC fonts as well as CM, etc. This *does* affect the rasterization to some extent, but in the cases I tried, it was no worse, and sometimes better. - - a small test file, modetest.tex, included. - - new modes for the Amiga ShowDVI previewer, PC previewing, high-resolution fax, LJ 5, Canon BJC, Lexmark Optra 4049 R, Agfa 3400PS. - - default localfont now ljfour. - - screen_rows, screen_cols interchanged to match plain.mf. - - TeX formatting improved, but still not completely working. As always, thanks to the contributors, and further additions and improvements are welcome. Please send bug reports or suggestions to tex-fonts@math.utah.edu (email tex-fonts-request to join the mailing list). General information: modes.mf is a collection of Metafont mode_def's. It also makes common definitions for write/white printers, `special' information, and landscape mode. It uses up too much memory for the table sizes in the original mf.web, so you either have to increase the sizes (as in Web2c) or rename the file and remove unneeded modes. I can't decipher mf.web well enough to understand how to make the modes use less memory; if some Metafont hacker can tell me, I'd very much like to hear it. If you have mode_def's which are not listed below, or corrections to the existing ones, please send them to me. Improvements to the exposition, particularly in how to create a new mode_def, are also welcome. kb@cs.umb.edu mode_def agfafzz = % AGFA 400PS mode_def agfatfzz = % AGFA P3400PS mode_def amiga = % Commodore Amiga mode_def aps = % Autologic APS-Micro5 mode_def apssixhi = % Autologic APS-Micro6 mode_def atariezf = % Atari ST SLM 804 printer mode_def atarinf = % Atari 95dpi previewer mode_def atarins = % Atari 96x96 previewer mode_def atariotf = % Atari ST SM 124 screen mode_def bitgraph = % BBN Bitgraph at 118dpi mode_def bjtenex = % Canon BubbleJet 10ex mode_def boise = % HP 2680A mode_def canonbjc = % Canon BJC-600, 360x360dpi mode_def canonex = % CanonEX in LaserWriter Pro 630 mode_def canonlbp = % e.g., Symbolics LGP-10 mode_def cg = % Compugraphic 8600 mode_def cgl = % Compugraphic 8600 landscape mode_def cgnszz = % Compugraphic 9600 mode_def crs = % Alphatype CRS mode_def cx = % Canon CX, SX, LBP-LX mode_def datadisc = % DataDisc mode_def newdd = % DataDisc with special aspect ratio mode_def declarge = % DEC 19-inch, 1280 x 1024 mode_def decsmall = % DEC 17-inch, 1024 x 768 mode_def deskjet = % HP DeskJet 500 mode_def docutech = % Xerox 8790 or 4045 mode_def dover = % Xerox Dover mode_def eighthre = % EightThree (83x83) mode_def epsdrft = % Epson at 120x72dpi mode_def epsdrftl = % Epson at 120x72dpi landscape mode_def epsfast = % Epson at 60x72dpi mode_def epsfastl = % Epson at 60x72dpi landscape mode_def epson = % 9-pin Epson MX/FX family mode_def epsonl = % 9-pin Epson MX/FX family landscape mode_def epsonact = % Epson Action Laser 1500 mode_def epsonlo = % Epson at 120x216dpi mode_def epsonlol = % Epson at 120x216dpi landscape mode_def epstylus = % Epson Stylus mode_def fourfour = % FourFour (44x44) (really low-res) mode_def gtfax = % 204 x 196dpi G3fax mode_def gtfaxl = % 204 x 196dpi G3fax landscape mode_def gtfaxlo = % 204 x 98dpi G3fax mode_def gtfaxlol = % 204 x 98dpi G3fax landscape mode_def highfax = % 200 x 200dpi G3fax mode_def hprugged = % HP RuggedWriter 480 mode_def ibm_a = % IBM 38xx (\#1) mode_def ibmd = % IBM 38xx (\#2) mode_def ibmega = % IBM EGA monitor mode_def ibmegal = % IBM EGA monitor landscape mode_def ibmfzon = % IBM 4019 mode_def ibmfztn = % IBM 4029-30, 4250 mode_def ibmpp = % IBM ProPrinter mode_def ibmppl = % IBM ProPrinter landscape mode_def ibmsoff = % IBM 6154 display mode_def sherpa = % IBM 6670 (Sherpa) mode_def ibmteot = % IBM 3812 mode_def ibmtetz = % IBM 3820 mode_def ibmtont = % IBM 3193 screen mode_def ibmtosn = % IBM 3179 screen mode_def ibmtosnl = % IBM 3179 screen landscape mode_def ibmvga = % IBM VGA monitor mode_def ibx = % Chelgraph IBX mode_def itoh = % CItoh 8510A mode_def itohl = % CItoh 8510A landscape mode_def itohtoz = % CItoh 310 mode_def itohtozl = % CItoh 310 landscape mode_def iw = % Apple ImageWriter mode_def jetiiisi = % HP Laser Jet IIISi mode_def lasf = % DEC LA75 mode_def lexmarkr = % IBM (Lexmark) Optra R (4049) mode_def linolo = % Linotype Linotronic [13]00 at 635dpi mode_def linolttz = % Linotype Linotronic L-300 with RIP-50 mode_def linoone = % Linotype Linotronic [13]00 at 1270dpi mode_def linotzzh = % Linotype Linotronic 300 at 2540dpi mode_def ljfive = % HP LaserJet 5 mode_def ljfour = % 600dpi HP LaserJet 4 mode_def ljlo = % HP LaserJet at 150dpi mode_def lmaster = % 1000dpi LaserMaster mode_def lnzo = % DEC LN01 mode_def lpstz = % DEC lps20 mode_def lqlores = % Epson LQ-500, 180x180dpi mode_def lqmed = % Epson LQ-500, 360x180dpi mode_def lqmedl = % Epson LQ-500, 360x180dpi landscape mode_def lview = % Sigma L-View monitor mode_def lwpro = % Apple LaserWriterPro 810 mode_def macmag = % Mac screens at magstep 1 mode_def mactrue = % Mac screens at 72dpi mode_def ncd = % NCD 19-inch mode_def nec = % NEC 180dpi mode_def nechi = % NEC-P6 at 360x360dpi mode_def neclm = % NEC PC-PR406LM 320dpi mode_def nectzo = % NEC PC-PR201 series 160dpi mode_def nexthi = % NeXT 400dpi, Newgen mode_def nextscrn = % 100dpi NeXT monitor mode_def nineone = % NineOne (91x91) mode_def nullmode = % TFM files only mode_def onetz = % OneTwoZero (120/120) mode_def onezz = % OneZeroZero (100x100) mode_def ocessfz = % OCE 6750-PS mode_def okidata = % Okidata mode_def okidatal = % Okidata landscape mode_def okifte = % Okidata 410e in 600 DPI mode mode_def pcscreen = % also, e.g., high-resolution Suns mode_def pcprevw = % preview on pc screen mode_def phaser = % Tektronix Phaser PXi mode_def prntware = % Printware 720IQ mode_def qms = % QMS (Xerox engine) mode_def qmsostf = % QMS 1725 mode_def qmsoszz = % QMS 1700 mode_def ricoh = % e.g., TI Omnilaser mode_def ricoha = % e.g., IBM 4216 mode_def ricohlp = % e.g., DEC LN03 mode_def sparcptr = % Sun SPARCprinter mode_def starnlt = % Star NL-10 mode_def starnltl = % Star NL-10 landscape mode_def sun = % Sun and BBN Bitgraph at 85dpi mode_def supre = % Ultre*setter at 2400dpi mode_def toshiba = % Toshiba 13XX, EpsonLQ mode_def ultre = % Ultre*setter at 1200dpi mode_def vs = % VAXstation monitor mode_def vtftzz = % Varityper 4200 B-P mode_def vtftzzhi = % Varityper 4300P at 2400dpi mode_def vtftzzlo = % Varityper 4300P at 1200dpi mode_def vtfzszw = % Varitype 5060W, APS 6 mode_def vtszz = % Varityper Laser 600 mode_def xrxesnz = % Xerox 8790 or 4045 mode_def xrxfzfz = % Xerox 4050/4075/4090/4700 mode_def xrxnszz = % Xerox 9700 mode_def xrxtszz = % Xerox 3700 ------------------------------ End of TeXhax Digest V96 #1 *************************** About TeXhax... Please send contributions to: TeXhax@tex.ac.uk Subscription and unsubscription requests: send a one line mail message to TeXhax-Request@tex.ac.uk containing either subscribe texhax or unsubscribe texhax If you have problems with un/subscribing, please mail owner-texhax@nott.ac.uk To obtain the Frequently Asked Questions (FAQ) lists for TeX, send a message with no subject to fileserv@shsu.edu, consisting of SENDME FAQ For information on the TeX Users Group, please send a message to TUG@TUG.org, or write TeX Users Group, 1850 Union Street, #1637 San Francisco CA 94123 (phone: 1 415 982 8449, fax: 1 415 982 8559) Backnumbers of all the digests are stored in the Comprehensive TeX Archive Network (CTAN) and can be retrieved on the Internet by anonymous ftp. The hosts comprising CTAN include, among others, ftp.dante.de (129.69.1.12) -- Germany ftp.shsu.edu (192.92.115.10) -- USA ftp.tex.ac.uk (128.232.1.87) -- UK Please use your nearest server, to keep network load down. The file /tex-archive/CTAN.sites on each of these hosts gives a list of other sites which maintain full or partial mirrors of the CTAN. Alternatively, finger ctan_us@ftp.shsu.edu for full details. TeXhax Digest back issues are filed below /tex-archive/digests/texhax/ Keyword-In-Context indexes are filed in /tex-archive/digests/indexes/ A Hypermail version of TeXhax is also available on the World-Wide Web at URL http://www.tex.ac.uk/tex-archive/digests/hyper/ \bye From owner-texhax-digest@nottingham.ac.uk Wed Jan 17 14:08:14 1996 Flags: 000000000001 Received: from jess.ccc.nottingham.ac.uk (jess.ccc.nottingham.ac.uk [128.243.40.193]) by csc-sun.math.utah.edu (8.7.1/8.7.1) with ESMTP id OAA20784 for ; Wed, 17 Jan 1996 14:07:48 -0700 (MST) Message-Id: <199601172107.OAA20784@csc-sun.math.utah.edu> Received: from nottingham.ac.uk by jess.ccc.nottingham.ac.uk id <06297-0@jess.ccc.nottingham.ac.uk>; Wed, 17 Jan 1996 15:54:48 +0000 From: Majordomo list server To: texhax-digest@nottingham.ac.uk Subject: TeXhax Digest V96 #2 Reply-To: TeXhax@tex.ac.uk Errors-To: owner-texhax-digest@nottingham.ac.uk Precedence: bulk Date: Wed, 17 Jan 1996 15:54:48 +0000 Sender: owner-texhax-digest@nottingham.ac.uk TeXhax Digest Wednesday, 17 January 1996 Volume 96 : Number 002 (incorporating UKTeX Digest) Today's Topics: TeX/LaTeX on Windows Re: LaTeX2e latex2e IEEEtran macro wanted. EmTeX printing to Canon BJ200ex printer gtex installation help TUGboat 16 #3 is being mailed New problems from a newcomer... ---------------------------------------------------------------------- From: "Sam Nelson (CO)" Date: Tue, 9 Jan 1996 10:27:33 +0000 (GMT) Subject: TeX/LaTeX on Windows Not being a beginner, this is going to sound a bit weird, I suppose, but despite having used TeX and LaTeX around here for more than a decade, I've never been asked (very much) to put them up on PC platforms, which are in little use in this department. However, I've just been asked about running LaTeX on a Windows platform, and it occurs to me that I know nothing whatever about this. Starting from scratch then, in an early-1996 Windows environment, where should one go to get the ideal TeX/LaTeX setup? I assume there are `commercial' solutions (we use Textures on Macintosh already) but I don't know how much money might be available, so a price spectrum from zero to lottery-winner is appropriate, I think. Thanks for any suggestions, Sam. - -- Sam Nelson, Comp Sci, Stirling U, FK9 4LA, Scotland ,->0->M Email: sam@cs.stir.ac.uk Pager: 0839 456640 I->3-+->2->R=->-+->4->O Tel: +44 1786 467443 Fax: +44 1786 464551 `->1->S=->-' URL: http://www.cs.stir.ac.uk/~sam R$+@$+ $@smtp$#$2$:$1@$2 ------------------------------ From: David Carlisle Date: Tue, 9 Jan 96 17:33:17 GMT Subject: Re: LaTeX2e > I currently possess just LaTeX, > ... > However, I notice that LaTeX2e is mentioned a great deal. What are > the advantages, if any, of switching to this? I think this question is the result of a misunderstanding. LaTeX2e is not an `alternative to LaTeX' but rather is just the current version of LaTeX. The previous version (2.09) is now obsolete. Specifically if you should decide to re-install a complete emtex on your PC (This is probably a good idea anyway, as emtex was significantly re-organised and improved earlier this year) then the LaTeX version that you will get by default will be LaTeX2e. emtex (including LaTeX, and suitable previewers etc) is available from ctan sites such as ftp.tex.ac.uk in tex-archive/systems/msdos/emtex. You can make a basic installation by following QUICK.ENG (quick English instructions) to be found in that directory. David ------------------------------ From: "Dr. Ke Chen" Date: Fri, 12 Jan 1996 17:11:18 +0800 (GMT+0800) Subject: latex2e IEEEtran macro wanted. I found the previous IEEEtran.sty did not work in LaTex2e. Some days ago, a person sent me his own new IEEEtran.sty but there are a few problems such as that the title of section is always typeset with a small font, etc. As a result, I would appreciate it if anyone could tell me where a complete IEEEtran macro in latex2e is available. Ke Chen #========================================================================# # Dr. Ke Chen, Associate Professor Tel: +86-10-2751935(O) # # National Lab of Machine Perception +86-10-2585703(H) # # The Center of Information Science Fax: +86-10-2563883,2552779 # # Peking University Email: chen@cis.pku.edu.cn # # Beijing 100871, China chenke@pku.edu.cn # #========================================================================# ------------------------------ From: maues@csv.warwick.ac.uk Date: Thu, 11 Jan 1996 20:51:00 +0000 (GMT) Subject: EmTeX printing to Canon BJ200ex printer I am absolutely a beginner as you can tell. I have successfully installed emTex onto my PC and now is the Printer. (use under DOS) I have a Canon BJ200ex printer. Q1. When the time I use the commands: 1) latex2e sample2e OKAY (where sample2e is the file name) 2) v @bj samples2e OKAY 3) prtbj10e samples2e ===> 6 blank pages 4) prtbj300 sample2e ==> 6 blank pages 5) prtp6h samples2e ===> there are some pages print out, which is the original BUT for each line of the orginal document, it comes out twice and into four separate parts. By the way, p6h is not the printer I am using, but every time I used the print command, a file for the p6h fonts created under the font directory. Do you know what is it about? And what should I do now? What command should I use? Thanks __\/__ . / ^ _ \ . |\| (o)(o) |/| #---------------.OOOo--oo--oOOO.--------------# # Trevor Jonathan Yuen # # Mathematics Institute # # University of Warwick # # Phone: (0973) 345 457 # #_________________________Oooo._______________# .oooO ( ) ( ) ) / \ ( (_/ \_) ------------------------------ From: Joao Luis Cardoso Soares Date: Mon, 15 Jan 1996 09:59:41 -0500 (EST) Subject: gtex installation help I've installed gtex 2.2 (by Young U. Ryu) on a Pentium machine running windows 95, but I keep getting two error messages which might be related: 1) After running \tex\set-tex.bat I get Sharing violation - c:\tex\set-tex.bat 2) When trying to open a DVI file whose fonts are to be created, I get Cannot find the file ". Make sure that the file exists on your system and that the path and filename are correct. Does anyone have a suggestion, or at least let me know the email address of the author. Many thanks, Joao - -- Joao Luis Cardoso Soares E-mail: jsoares@groucho.gsb.columbia.edu Columbia University Tel : (212) 749-6222 (Home) 804 Uris Hall Fax : (212) 864-4857 (PhD student) New York, NY 10027 http://www.cc.columbia.edu/~jls55/ ------------------------------ From: bbeeton Date: Tue, 16 Jan 1996 17:44:10 -0500 (EST) Subject: TUGboat 16 #3 is being mailed i'm pleased to announce that tugboat 16#3 will be mailed from the printer tomorrow. the table of contents for this issue is attached. tugboat 16#4 will be sent to the printer within the next few days. the schedule calls for mailing three weeks after the printer receives the copy. the next issue, 17#1, will be dated march 1996 and should be delivered to the printer within that month. thanks to all the members of the production team who have made very great efforts to get this back on schedule. -- barbara beeton -------------------- TUGboat Volume 16, Number 3 / September 1995 ================================ Contents (128 pages) Robin Fairbairns Production notes 222 Opening Address Michel Goossens President's words 223 Fonts Jiri Zlatuska 227 When MF does it alone Richard J. Kinch MetaFog: converting MF shapes to contours 233 Alan Hoenig The Poetica family: fancy fonts with TeX and LaTeX 244 Michel Goossens Using Adobe Type 1 Multiple Master fonts with TeX 253 Jeremy Gibbons Dotted and dashed lines in MF 259 Sergey Lesenko Printing TeX documents with partial Type 1 fonts 265 LaTeX Matthew Swift Modularity in LaTeX 269 Dennis Kletzing A multienumerate package 276 Hyphenation Petr Sojka and Pavel Sevecek Hyphenation in TeX --- Quo Vadis? 280 Petr Sojka Notes on compound word hyphenation in TeX 290 Literate programming Wlodek Bzyl Literate Plain source is available! 297 Bart Childs, Deborah Dunn and William Lively Teaching CS/1 courses in a literate manner 300 Methods T.V. Raman An audio view of (La)TeX documents --- part II 310 Sebastian Rahtz Another look at LaTeX to SGML conversion 315 Robin Fairbairns Omega --- Why bother with Unicode? 325 Gabriel Valiente Feruglio Modern Catalan typographical conventions 329 News & Announcements TUG'96 Announcement 339 Calendar 340 TUG Business TUG'95 --- List of Attendees 341 Institutional members 344 Advertisements TeX consulting and production services 345 %% END OF FILE ------------------------------ From: gabriele@dtovf1.roma2.infn.it (gabriele migliorini) Date: Wed, 17 Jan 96 13:12:34 +0100 Subject: New problems from a newcomer... Dear readers, at the very beginning, I have to thank all of you for reading my previous post, an trying to give me sensible replies. To summarize, the best answer to the bold greek letters is to simply get and run amsbsy, that's part of the amslatex distribution. It gives two new commands: a \boldsymbol that embolden a char in a font having a correspondent bold-face, and a 'ppor men's bold', that overstrikes a normal font, emboldening it even if there is not a bold typeface for printing it. As I got asmlatex, I got many new useful functions, like an extendable arrow symbol for writing limits, and a couple of new strange (?bugs?): in the manual, I read that there are a couple of operator redefining commands, that should make a log-like operator, with eventual _{text} printed at bottom right of the operator, and a lim like one, with text printed under the operator. But, in my printed version, they look exactly the same, and therefore, something is not doing it job. Furthermore, when I latexed the testmath.tex document, I got some \i command undefined in math mode, about lines containing a \begin or a \end... Are those signs of an incompatible behavior of a) the latex release of december'95 and amslatex b) TeX 3.1415 and the other programs c) the test documents, and amslatex (sure enough, I'm joking here!) Now, I'm crying for help another time: You know, I was writing down some lecture notes for a course I attended In march, and I started using latex after a disaster with the copy of word on the computer I used: the equation editor suddenly decided not to run anymore. Now, I have half of my work in Latex, and another half in Word 6 format. Is there anybody who does know how to convert the latter? I read, in the documentation about programs on ctan, that such a program cannot be public domain... But does there exist a commercial program doing the job? If it does exist, does some of you own a copy of it, and can be so nice to let me send him my document, and convert it for me? I'll be, as usual, grateful 'till the end of my life to anybody helping me. Sincerely, l. ------------------------------ End of TeXhax Digest V96 #2 *************************** About TeXhax... Please send contributions to: TeXhax@tex.ac.uk Subscription and unsubscription requests: send a one line mail message to TeXhax-Request@tex.ac.uk containing either subscribe texhax or unsubscribe texhax If you have problems with un/subscribing, please mail owner-texhax@nott.ac.uk To obtain the Frequently Asked Questions (FAQ) lists for TeX, send a message with no subject to fileserv@shsu.edu, consisting of SENDME FAQ For information on the TeX Users Group, please send a message to TUG@TUG.org, or write TeX Users Group, 1850 Union Street, #1637 San Francisco CA 94123 (phone: 1 415 982 8449, fax: 1 415 982 8559) Backnumbers of all the digests are stored in the Comprehensive TeX Archive Network (CTAN) and can be retrieved on the Internet by anonymous ftp. The hosts comprising CTAN include, among others, ftp.dante.de (129.69.1.12) -- Germany ftp.shsu.edu (192.92.115.10) -- USA ftp.tex.ac.uk (128.232.1.87) -- UK Please use your nearest server, to keep network load down. The file /tex-archive/CTAN.sites on each of these hosts gives a list of other sites which maintain full or partial mirrors of the CTAN. Alternatively, finger ctan_us@ftp.shsu.edu for full details. TeXhax Digest back issues are filed below /tex-archive/digests/texhax/ Keyword-In-Context indexes are filed in /tex-archive/digests/indexes/ A Hypermail version of TeXhax is also available on the World-Wide Web at URL http://www.tex.ac.uk/tex-archive/digests/hyper/ \bye From texhax-digest-outgoing-request@nottingham.ac.uk Thu Mar 7 15:07:58 1996 Flags: 000000000001 Received: from jess.ccc.nottingham.ac.uk (jess.ccc.nottingham.ac.uk [128.243.40.193]) by csc-sun.math.utah.edu (8.7.4/8.7.3) with ESMTP id PAA16431 for ; Thu, 7 Mar 1996 15:04:39 -0700 (MST) Message-Id: <199603072204.PAA16431@csc-sun.math.utah.edu> Received: from nottingham.ac.uk by jess.ccc.nottingham.ac.uk id <26648-0@jess.ccc.nottingham.ac.uk>; Thu, 7 Mar 1996 17:29:03 +0000 From: Majordomo list server To: texhax-digest@nottingham.ac.uk Subject: TeXhax Digest V96 #3 Reply-To: TeXhax@tex.ac.uk Errors-To: owner-texhax-digest@nottingham.ac.uk Precedence: bulk Date: Thu, 7 Mar 1996 17:29:03 +0000 Sender: owner-texhax-digest@nottingham.ac.uk TeXhax Digest Thursday, 7 March 1996 Volume 96 : Number 003 (incorporating UKTeX Digest) Today's Topics: TeX/LaTeX on Windows Re: TeXhax Digest V96 #2 Re: TeXhax Digest V96 #2 spell checker for sun(UNIX) permille, fractions, space, \afterpage and procs Table of contents, TUGboat 16(2) Announcing TUG'96 in Dubna (Russia) July 28-Aug 2 1996 ---------------------------------------------------------------------- From: bkph@ai.mit.edu (Berthold K.P. Horn) Date: Wed, 17 Jan 1996 16:11:04 -0500 Subject: TeX/LaTeX on Windows From: "Sam Nelson (CO)" Sam Nelson, Comp Sci, Stirling U, FK9 4LA, Scotland ,->0->M > Email: sam@cs.stir.ac.uk Pager: 0839 456640 I->3-+->2->R=->-+->4->O > Tel: +44 1786 467443 Fax: +44 1786 464551 `->1->S=->-' > URL: http://www.cs.stir.ac.uk/~sam R$+@$+ $@smtp$#$2$:$1@$2 DISCLAIMER: respondent has connections with Y&Y ------------------------------ From: Jonathan Dixon Date: Fri, 19 Jan 96 13:59:29 MST Subject: Re: TeXhax Digest V96 #2 You wrote: > From: "Dr. Ke Chen" > Date: Fri, 12 Jan 1996 17:11:18 +0800 (GMT+0800) > Subject: latex2e IEEEtran macro wanted. > > I found the previous IEEEtran.sty did not work in LaTex2e. Some days ago, > a person sent me his own new IEEEtran.sty but there are a few problems > such as that the title of section is always typeset with a small font, > etc. As a result, I would appreciate it if anyone could tell me where > a complete IEEEtran macro in latex2e is available. > > Ke Chen > > #========================================================================# > # Dr. Ke Chen, Associate Professor Tel: +86-10-2751935(O) # > # National Lab of Machine Perception +86-10-2585703(H) # > # The Center of Information Science Fax: +86-10-2563883,2552779 # > # Peking University Email: chen@cis.pku.edu.cn # > # Beijing 100871, China chenke@pku.edu.cn # > #========================================================================# I have a version I modified to make an IEEEtran.cls. I haven't tested it extensively, but for what I've done it seems to work. If someone can provide a place to post it to, I'd be more than willing to have it distributed. - -- Jon Dixon dixonj@colorado.edu http://spot.colorado.edu/~dixonj/ ------------------------------ From: "Dr. Ke Chen" Date: Mon, 22 Jan 1996 09:20:40 +0800 (GMT+0800) Subject: Re: TeXhax Digest V96 #2 On Fri, 19 Jan 1996, Jonathan Dixon wrote: > Date: Fri, 19 Jan 96 13:59:29 MST > From: Jonathan Dixon > To: TeXhax@tex.ac.uk > Cc: chen@cis.pku.edu.cn > Subject: Re: TeXhax Digest V96 #2 > > You wrote: > > From: "Dr. Ke Chen" > > Date: Fri, 12 Jan 1996 17:11:18 +0800 (GMT+0800) > > Subject: latex2e IEEEtran macro wanted. > > > > I found the previous IEEEtran.sty did not work in LaTex2e. Some days ago, > > a person sent me his own new IEEEtran.sty but there are a few problems > > such as that the title of section is always typeset with a small font, > > etc. As a result, I would appreciate it if anyone could tell me where > > a complete IEEEtran macro in latex2e is available. > > > > Ke Chen > > > I have a version I modified to make an IEEEtran.cls. I haven't tested it > extensively, but for what I've done it seems to work. If someone can > provide a place to post it to, I'd be more than willing to have it distributed. > > -- > Jon Dixon > dixonj@colorado.edu > http://spot.colorado.edu/~dixonj/ > Thanks very much for your kind response. I think you might upload it to CTAN archives. In addition, I would appreciate it if you would send this macro to me by email for my immediate use. Ke Chen #========================================================================# # Dr. Ke Chen, Associate Professor Tel: +86-10-2751935(O) # # National Lab of Machine Perception +86-10-2585703(H) # # The Center of Information Science Fax: +86-10-2563883,2552779 # # Peking University Email: chen@cis.pku.edu.cn # # Beijing 100871, China chenke@pku.edu.cn # #========================================================================# ------------------------------ From: dhlee@pearl.cs.pusan.ac.kr (DoHoon Lee) Date: Tue, 30 Jan 96 19:02:12 KST Subject: spell checker for sun(UNIX) I am looking for spell checker utility for LaTeX. Does anybody know where this be found ? Or If you have spell checker for LaTeX, please let me got it. Thanks in advance, Dohoon Lee ============================================================ Dept. of Computer Science Pusan National University Pusan 609-735, Korea dhlee@pearl.cs.pusan.ac.kr ============================================================ ------------------------------ From: Kris Lockyear Date: Tue, 30 Jan 1996 16:17:08 PST Subject: permille, fractions, space, \afterpage and procs Could anyone help me with five queries? The first two are perhaps simple, the last two I am unsure about. FYI I am using LaTeX2e with emTeX on a PC. 1) How can I get a permille sign (like a percent sign but with two zeros -- 0/00). I scanned the tables in The Companion and The Book but don't see it. 2) How can I get nice in-text fractions? By that I mean not 1/2 which $1/2$ would give you, or \frac{1}{2}, but a fraction where there is a small 1, a diagonal line and a small 2. (I usually use the Times package and a PS laserprinter). 3) I have a chapter which is nearly all text with some headings/sub-headings and footnotes. In places, LaTeX has increased the white space between paragraphs or around sub-headings in order to place a \section at the top of a page. This looks very ugly and I would rather have a little extra white space at the bottom of the page, than extra white space between paragraphs. Is there any way I can stop LaTeX adding this extra space? 4) In another chapter I have a lot of tables and figures. Following advice in The Companion, I often use \afterpage{\clearpage} to flush out the figures etc. Unfortunately, this occasionally results in a blank page with the header at the top, and a footnote at the bottom. Any suggestions? 5) Lastly, later this year I have to produce a set of conference proceedings. Could I have some pointers to classes/packages and/or articles which would enable me to do this in LaTeX2e. Specifically, including authors names and addresses in paper titles and the TOC etc. Many thanks in advance, Kris Lockyear. ------------------------------ From: Robin Fairbairns Date: Thu, 01 Feb 1996 14:36:53 +0000 Subject: Table of contents, TUGboat 16(2) It appears that we (the TUGboat production team) did not post this table of contents. Apologies to all ... TUGboat Volume 16, Number 2 / June 1995 Guest Editor: Malcolm Clark ================================ Addresses 99 General Delivery Michel~Goossens 101 Opening words Malcolm~Clark Introduction 101 Theme Issue on Portability of Electronic Documents Michel~Goossens and Janne~Saarela A practical introduction to SGML 103 Peter~Flynn HTML & TeX: Making them sweat 146 Geeti~Granger The inside story of life at Wiley with SGML, 151 LaTeX and Acrobat Mark~D.~Doyle The Los Alamos e-print archives: HyperTeX in action 154 Otfried~Schwarzkopf The Hyperlatex story 159 Yannis~Haralambous and Sebastian~Rahtz LaTeX, hypertext and PDF, or the entry of TeX into the world of hypertext 162 Michel~Goossens and Janne~Saarela TeX to HTML and back 174 News & Announcements Calendar 215 Late-Breaking News Mimi~Burbank and Barbara Beeton Production notes 100 Coming next issue TUG Business Institutional members 217 Forms TUG membership application Advertisements TeX consulting and production services 218 Index of advertisers 218 %%% end of file ------------------------------ From: Michel Goossens Date: Wed, 7 Feb 1996 00:44:11 +0100 (MET) Subject: Announcing TUG'96 in Dubna (Russia) July 28-Aug 2 1996 ====================================================================== The 17th Annual TeX Users Group Meeting polyglot polytechnic polymath POLY-TeX polymath polytechnic polyglot July 28-- August 2, 1996 in JINR, Dubna, Russia Organized jointly by: TUG, CyrTUG, JINR CALL FOR PAPERS ====================================================================== Dear TeX Friends! So the time has arrived when TeX, the Polyglot, who has for many years happily ``spoken'' many different languages written in the Latin alphabet, extends its knowledge in the field of other alphabets. During the Summer of 1996, a visit is planned to Russia where TeX will have its first practical session in Cyrillic. Russia is that large country, with its enormous spaces, inhabited by those enigmatic Russians, who started the century with a Revolution, and ended it with ``Perestroyka''. A country who founded a brilliant mathematical school, colonized the Cosmos, and conquered the world with its literature, music, and ballet. So what awaits the TeX user who plans to attend TUG'96? At Moscow's international Sheremetevo-2 Airport, conference participants will be met by a member of the organizing committee and escorted by bus to Dubna, where from July 28 to August 2, the 17th TUG conference will take place (see below for more about Dubna). If, for some reason or another, you envision cold, the taiga, and white bears, then we can reassure you, Dubna is in the European part of Russia, and if you want to see the ice on the nordic oceans, you will have to travel further than you would to reach the subtropics of the Black Sea shore. Indeed, you will soon find that the pine forest in which the comfortable town of Dubna is situated will remind you of a park. In July and August, the weather is mostly warm and sunny, with an average temperature of 28 degrees Centigrade. Guests will be housed in a comfortable hotel on the banks of the Volga river in single or double rooms (hot water, shower, telephone, and television). The Russian ``cuisine'' is characterized by its abundance, so one can forget about slimming. The social program includes a picnic on the picturesque banks of the Volga, where we will be taken by boat, a bus excursion to Sergiev Posad (the center of the Russian Orthodox Church, where Andrey Slephkhin works--- see his article on page 373 of TUGboat 16#4 or the EuroTeX'95 proceedings page 331), and, on the last day a visit to Moscow, following which the participants can be dropped of at the airport to fly home, or at one of the railway stations, if they want to prolong their visit. The conference's preliminary program will be announced as soon as we receive from you, dear readers, proposals for presentations, courses that you would like to teach or attend, poster sessions, or any problem(s) or subject(s) that interest you. Please send your suggestions to the conference electronic address TUG96@pds.jinr.ru ================= The theme of the Conference is: PolyTeX, TeX or the art of multi-lingual, maths and technical typesetting. polyglot polytechnic polymath POLY-TeX polymath polytechnic polyglot =================== PROGRAM COMMITTEE =================== Evgeniy Vasilievitch Pankratiev Moscow, Russia Email: pankrat@shade.msu.ru Michel Goossens Geneva, Switzerland Email: goossens@cern.ch Mimi Burbank Florida, USA Email: mimi@scri.fsu.edu =========== DEADLINES =========== Submission of abstracts February 20 <<<<<<< Acceptance signified to authors February 29 <<<<<<< Preliminary articles March 31 <<<<<<< Proposals for workshops, demos, and poster sessions April 20 Registration and transfer of a non-refundable sum of $100 per person to a Dubna bank May 31 Visa supporting information June 5 Revised articles June 10 <<<<<<< Start of Conference July 28 ====================== ORGANIZING COMMITTEE ====================== Vladimir Vasilievitch Korenkov Dubna, Russia Email: korenkov@cv.jinr.ru Irina Anatolievna Makhovaya Moscow, Russia Email: irina@mir.msk.su Sebastian Rahtz Oxford, UK Email: s.rahtz@elsevier.co.uk ====================== PRACTICAL INFORMATION ======================= - - ---------------- Conference costs - - ---------------- The Conference Committee foresees a cost in the range 550--600 USD. This sum includes the complete cost of the conference, namely the registration fee, lodging (6 nights with six breakfasts, lunches, and dinners), coffee/tea breaks, social events, and transport from Sheremetevo Airport to Dubna. The payment should be made in the following way: a `non-refundable' sum of $100 per person should be transferred to a bank account (to be announced) before June 1st. After receipt of that sum an official invitation, necessary for obtaining a visa (see below), will be faxed to the participant. The rest will be payable in cash upon arrival at the Conference (no credit cards or cheques can be used in Dubna). %For participants of economically weaker countries or for students %we hope to arrange partial support via sponsors. We hope to arrange bursary funds for support of students and those participants who demonstrate need. - - ----- Visas - - ----- Most of the visitors from outside Russia will need a visa to attend the Conference. Therefore, for arranging a visa into Russia, participants should inform the Mrs. N. Dokalenko, Conference Secretariat E-mail nataly@ypr.jinr.dubna.su; Fax 7 095 975 2381 or 7 09621 65 891 of their and (possibly) the accompanying person(s)'s full name, date of birth, citizenship, passport number, arrival and departure dates. The Secretariat will forward by fax the visa support message to the participants with which they should apply for visas to the nearest Russian Embassy or Consulate. Please note that you should apply for a visa valid for Dubna, Moscow, and Sergiev Posad. (If you plan to visit other cities in Russia you should obtain the relevant documents and join them to the visa application, so that the names of all places to be visited can be entered on the visa form as required.) - - -------------- Transportation - - -------------- The Organizing Committee will arrange direct transportation by bus >From the Sheremetevo-2 Airport to Dubna (130 km north of Moscow). The Secretariat should be informed of the flight number, precise date and time of arrival (Moscow time) `no later than' four working days before a participant wishes to be met at the airport. It is our intention to have each participant met by a member of Organizing Committee. Details will be available later. ==================== WELCOME TO DUBNA ! ==================== Dubna was founded in 1956 when the Convention establishing the Joint Institute for Nuclear Research was signed. The town is situated on the picturesque banks of the Volga river and the Moscow sea 120 km to the north of Moscow. One can reach Dubna from Moscow within 2 hours going by car, by bus or by express train. It will take you 1.5 hours to go to Dubna from the Sheremetevo-2 International Airport. Waterways connect Dubna not only to the Russian Volga cities, but also to the waters of the Black, the Caspian, the Baltic and the White seas. There is no harmful environmental impact from the industrial plants; this together with the large tracts of forest in the environs of Dubna, and the vast water area dotted with small islands, makes the area quite attractive for tourism and rest. The Volga embankment is one of the prettiest parts of the town. In springtime, the streets of Dubna are full of the odour of lilacs, the apple trees are pink-white; in summer, lime trees, maples, birch trees and poplars make the town seem totally green; in autumn the town is all golden excepting the evergreen of old pine trees. The town's modern look harmonizes with the quietness of the surrounding forest. The town was built in the midst of a forest. There are separate patches of trees in the town itself, and the town park is just a part of the forest. It takes just a few minutes to get to the forest from the shopping centre on foot. A few minutes' walk and you are outside the city limits! Small as it is, Dubna is a real metropolis. It is a scientific metropolis. It is a ``big little city'', as a visiting American scientist called it many years ago. Since the foundation of the Joint Institute for Nuclear Research (JINR), the name of Dubna has constantly been in the pages of the world's newspapers and journals. Dubna is one of the world centres for fundamental research in nuclear physics. The Joint Institute plays an important role as a coordinator of investigations of the scientists from 18 JINR member-state institutes. Wide international scientific and technical cooperation is one of the fundamental concepts of the JINR. Dubna is indeed a town of international friendship. Foreign speech can be heard everywhere. But the words, no matter in which language they are pronounced, are clear to everybody: friendly cooperation and fraternity unite all the physicists and mathematicians living and working in Dubna into an international scientific community. On October 3, 1994, Dubna opened its doors to its first university, ``International University of Dubna: Nature, Society and Man''. The university is composed of five "cathedra" or faculties, including socioeconomic sciences, ecology and earth science, computer education, linguistics, and health and physical education. Two more faculties--- law and government and technology--- are also being contemplated. The town has great experience in holding international conferences, and exchanges of delegations between countries in the sphere of science, education and culture. Dubna and La Crosse, Wisconsin, USA, are sister cities. Dubna is famous for its hospitality. Famous scientists, public figures and statesmen from different countries visit Dubna. They are always impressed by the gracious welcome they receive in Dubna and warm generosity which Dubna residents demonstrate. ======================= A FEW WORDS ON MOSCOW ======================= Moscow, the capital of Russia, has a population of some nine million people. It is a city rich in cultural, architectural and historical monuments, and, at the same time, boasts a rapidly developing modern urban community with brand new blocks of flats, long, straight and broad avenues, parks, gardens, stadiums, schools, cinemas, department stores, recreation centres, bridges and highways. Though forward-looking, it cherishes the memory of its past, and its old sections lend it a special charm. Michel Goossens / TUG President ------------------------------ End of TeXhax Digest V96 #3 *************************** About TeXhax... Please send contributions to: TeXhax@tex.ac.uk Subscription and unsubscription requests: send a one line mail message to TeXhax-Request@tex.ac.uk containing either subscribe texhax or unsubscribe texhax If you have problems with un/subscribing, please mail owner-texhax@nott.ac.uk To obtain the Frequently Asked Questions (FAQ) lists for TeX, send a message with no subject to fileserv@shsu.edu, consisting of SENDME FAQ For information on the TeX Users Group, please send a message to TUG@TUG.org, or write TeX Users Group, 1850 Union Street, #1637 San Francisco CA 94123 (phone: 1 415 982 8449, fax: 1 415 982 8559) Backnumbers of all the digests are stored in the Comprehensive TeX Archive Network (CTAN) and can be retrieved on the Internet by anonymous ftp. The hosts comprising CTAN include, among others, ftp.dante.de (129.69.1.12) -- Germany ftp.shsu.edu (192.92.115.10) -- USA ftp.tex.ac.uk (128.232.1.87) -- UK Please use your nearest server, to keep network load down. The file /tex-archive/CTAN.sites on each of these hosts gives a list of other sites which maintain full or partial mirrors of the CTAN. Alternatively, finger ctan_us@ftp.shsu.edu for full details. TeXhax Digest back issues are filed below /tex-archive/digests/texhax/ Keyword-In-Context indexes are filed in /tex-archive/digests/indexes/ A Hypermail version of TeXhax is also available on the World-Wide Web at URL http://www.tex.ac.uk/tex-archive/digests/hyper/ \bye From texhax-digest-outgoing-request@nottingham.ac.uk Mon Mar 11 15:51:48 1996 Flags: 000000000001 Received: from jess.ccc.nottingham.ac.uk (jess.ccc.nottingham.ac.uk [128.243.40.193]) by csc-sun.math.utah.edu (8.7.4/8.7.3) with ESMTP id PAA07408 for ; Mon, 11 Mar 1996 15:51:35 -0700 (MST) Message-Id: <199603112251.PAA07408@csc-sun.math.utah.edu> Received: from nottingham.ac.uk by jess.ccc.nottingham.ac.uk id <03918-0@jess.ccc.nottingham.ac.uk>; Mon, 11 Mar 1996 16:12:56 +0000 From: Majordomo list server To: texhax-digest@nottingham.ac.uk Subject: TeXhax Digest V96 #4 Reply-To: TeXhax@tex.ac.uk Errors-To: owner-texhax-digest@nottingham.ac.uk Precedence: bulk Date: Mon, 11 Mar 1996 16:12:56 +0000 Sender: owner-texhax-digest@nottingham.ac.uk TeXhax Digest Monday, 11 March 1996 Volume 96 : Number 004 (incorporating UKTeX Digest) Today's Topics: UKTUG Meeting: March 20 1996 --- TeX and the Internet revised dealines for TUG'96 Memo Class? startsection ---------------------------------------------------------------------- From: David Carlisle Date: Wed, 21 Feb 1996 15:07:21 GMT Subject: UKTUG Meeting: March 20 1996 --- TeX and the Internet The following message is a courtesy copy of an article that has been posted as well. UKTUG is holding a one day meeting at Warwick on `TeX and the Internet' Speakers include: Michel Goossens: LaTeX2HTML Malcolm Clark HTML3 maths Yannis Haralambous (To be announced) Robin Fairbairns The CTAN Archives Sebastian Rahtz Acrobat, TeX and the Web Details and a booking form are available on WWW http://www.ma.man.ac.uk/~carlisle/form2.html (They are in the post to all UKTUG members already.) David Carlisle For UKTUG ------------------------------ From: Michel Goossens Date: Tue, 20 Feb 1996 17:16:38 +0100 (MET) Subject: revised dealines for TUG'96 ============================ REVISED DEADLINES for TUG'96 ============================ Submission of abstracts March 11 << new date Acceptance signified to authors March 20 << new date Preliminary articles April 12 << new date Proposals for workshops, demos, and poster sessions April 20 Registration and transfer of a non-refundable sum of $100 per person to a Dubna bank May 31 Visa supporting information June 5 Revised articles June 10 << firm date Start of Conference July 28 ====================================================================== Please send your proposals for contributions to the TUG'96 Organizing Committee at their email address TUG96@pds.jinr.ru. The date of June 10th is a "firm date", since we want to have all articles in a final form ready to be printed by the time of the Conference, and therefore need them to go to the printer by late June. As a consequence final papers not received by June 10 will not appear in the Proceedings Issue of TUGboat. The most up-to-date information about TUG'96 will be available at the URL: http://www.scri.fsu.edu/~mimi/tug96/tug96.html or, by ftp from the CTAN archives in tex-archive/usergrps/tug/tug96/news-mon.asc The TUG'96 Organizing Committee ------------------------------ From: rpreuss@bbn.com (Rob Preuss) Date: Fri, 8 Mar 1996 17:07:58 -0500 Subject: Memo Class? Q: Regarding Memorandum Class(es) for LaTeX2e. (short form) Is there a popular/standard document class for creating memos with LaTeX2e (and the AMS-LaTeX 1.2 add-ons) using TeXtures? (long form) 10 years ago I wrote my own "memo" style using the standard "article" style as a starting point. Main features included: - double column option - 10pt, 11pt, 12pt options - titlesection (created by \maketitle) sorta like MEMORANDUM To: ... From: ... Subject: ... Date: ... Copies To: ... _______________________ - RHS header with + short form contents of the "To:" field, + contents of "Date:" field, and + Page x of xx. - footer with copyright info. Its been great, but now I'd like to upgrade to LaTeX2e while taking advantage of all the nifty stuff offered by AMS-LaTeX. 1. Do I have to reinvent this or does something similar exist? 2. If so, should I write a new class or just a package? 3. Should this new class/package be more related to the "article" class or the "amsart" class? Thanks very much! ______________ Dr. Robert D. Preuss (Rob) BBN Acoustic Technologies Voice: 617-873-3773 70 Fawcett Street Fax: 617-873-2894 or 2918 Cambridge, MA 02138 USA Internet: rpreuss@bbn.com ------------------------------ From: Mr Marcilo Alves Date: Mon, 11 Mar 1996 11:26:32 +0000 (GMT) Subject: startsection Hi, I'm trying to redefine the \section, \chapter look by using the \@startsection command (Companion, page 24). I always get the message you can't use \spacefactor in vertical mode Coul you help me? Here goes my input \documentclass{book} \renewcommand{\section}{\@startsection {\section}% {1}% {2mm}% {2mm}% {2mm} {\itshape}}% \begin{document} \section{Hi} This is ... \end{document} and here the message This is TeX, Version 3.1415 (C version 6.1) (lixo.tex LaTeX2e <1994/12/01> patch level 1 (/apps/TeX/inputs/tex/latex2e/base/book.cls Document Class: book 1994/12/09 v1.2x Standard LaTeX document class (/apps/TeX/inputs/tex/latex2e/base/bk10.clo)) No file lixo.aux. ! You can't use `\spacefactor' in vertical mode. \@->\spacefactor \@m l.10 \section {Hi} ? Thanks marcilio ------------------------------ End of TeXhax Digest V96 #4 *************************** About TeXhax... Please send contributions to: TeXhax@tex.ac.uk Subscription and unsubscription requests: send a one line mail message to TeXhax-Request@tex.ac.uk containing either subscribe texhax or unsubscribe texhax If you have problems with un/subscribing, please mail owner-texhax@nott.ac.uk To obtain the Frequently Asked Questions (FAQ) lists for TeX, send a message with no subject to fileserv@shsu.edu, consisting of SENDME FAQ For information on the TeX Users Group, please send a message to TUG@TUG.org, or write TeX Users Group, 1850 Union Street, #1637 San Francisco CA 94123 (phone: 1 415 982 8449, fax: 1 415 982 8559) Backnumbers of all the digests are stored in the Comprehensive TeX Archive Network (CTAN) and can be retrieved on the Internet by anonymous ftp. The hosts comprising CTAN include, among others, ftp.dante.de (129.69.1.12) -- Germany ftp.shsu.edu (192.92.115.10) -- USA ftp.tex.ac.uk (128.232.1.87) -- UK Please use your nearest server, to keep network load down. The file /tex-archive/CTAN.sites on each of these hosts gives a list of other sites which maintain full or partial mirrors of the CTAN. Alternatively, finger ctan_us@ftp.shsu.edu for full details. TeXhax Digest back issues are filed below /tex-archive/digests/texhax/ Keyword-In-Context indexes are filed in /tex-archive/digests/indexes/ A Hypermail version of TeXhax is also available on the World-Wide Web at URL http://www.tex.ac.uk/tex-archive/digests/hyper/ \bye From texhax-digest-outgoing-request@nottingham.ac.uk Mon Apr 15 11:55:28 1996 Flags: 000000000001 Received: from jess.ccc.nottingham.ac.uk (jess.ccc.nottingham.ac.uk [128.243.40.193]) by csc-sun.math.utah.edu (8.7.4/8.7.3) with ESMTP id LAA13034 for ; Mon, 15 Apr 1996 11:54:51 -0600 (MDT) Message-Id: <199604151754.LAA13034@csc-sun.math.utah.edu> Received: from nottingham.ac.uk by jess.ccc.nottingham.ac.uk id <27588-0@jess.ccc.nottingham.ac.uk>; Mon, 15 Apr 1996 11:52:28 +0100 From: Majordomo list server To: texhax-digest@nottingham.ac.uk Subject: TeXhax Digest V96 #5 Reply-To: TeXhax@tex.ac.uk Errors-To: owner-texhax-digest@nottingham.ac.uk Precedence: bulk Date: Mon, 15 Apr 1996 11:52:28 +0100 Sender: owner-texhax-digest@nottingham.ac.uk TeXhax Digest Monday, 15 April 1996 Volume 96 : Number 005 (incorporating UKTeX Digest) Today's Topics: Re: fractions, space, \afterpage (TeXhax) Re: startsection (TeXhax Digest V96 #4) Plain TeX query disc set and printer driver for LaTex 2e ---------------------------------------------------------------------- From: Donald Arseneau Date: Thu, 07 Mar 1996 20:47:44 PST Subject: Re: fractions, space, \afterpage (TeXhax) In TeXhax Digest V96 #3, Kris Lockyear wrote: % 2) How can I get ... 1/2 with small 1, a diagonal line and a small 2. It is in The TeXbook, exercise 11.6. You probably have manmac.tex and/or eplain.tex on your system somewhere; you can copy the definition out of there to avoid re-typing it. % 3) LaTeX increased white space between paragraphs to place a \section at the % top of a page. This looks very ugly. Declare \raggedbottom (described in the LaTeX manual) % 4) I often use \afterpage{\clearpage} to flush out the figures etc. % this occasionally results in a blank page with a footnote at the bottom. a) In my opinion, this is never necessary. It only helps when figures are *stuck* with impossible placement requirements. You are better off giving realistic placement options for your figures and setting higher values for \topfraction and \bottomfraction (say 0.8) and a lower \textfraction (0.2). Re-read the LaTeX manual discussion of these. b) Don't write long footnotes. c) You can avoid most split footnotes using: \raggedbottom \addtolength{\topskip}{0pt plus 1cm minus 0.2pt} (No, the top won't be ragged; just the bottom.) Donald Arseneau asnd@reg.triumf.ca ------------------------------ From: Robin Fairbairns Date: Tue, 12 Mar 1996 09:33:00 +0000 Subject: Re: startsection (TeXhax Digest V96 #4) Mr Marcilo Alves writes: >I'm trying to redefine the \section, \chapter look by >using the \@startsection command (Companion, page 24). Having not read the Companion, page 16 (I think it is), where it tells you to: \makeatletter >\renewcommand{\section}{\@startsection > {\section}% > {1}% > {2mm}% > {2mm}% > {2mm} > {\itshape}}% \makeatother This is one of the most-FAQs. The Companion is _not_ a beginner's book; if you've never programmed LaTeX macros before *PLEASE* (everyone, not just Mr. Alves!) read the introductory pages first (there are only twenty or so of them). ------------------------------ From: cgm@ssci.liv.ac.uk (Colin Mason) Date: Thu, 28 Mar 1996 18:47:11 GMT Subject: Plain TeX query % Could someone please answer a question which has been puzzling me for some % while because, unless I misunderstand the TeXbook, TeX is not doing what I % think it should do. Consider the following input file: %------------------------------------------------------------------------------- \chardef\return=`\^^M % \def\dopling{% \ifcat\space\noexpand\next \wlog{}% \else \ifcat\return\noexpand\next \wlog{}% \else \ifcat\bgroup\noexpand\next \wlog{}% \else \wlog{}% \fi \fi \fi}% % \catcode`\!=\active % \def!{\futurelet\next\dopling}% % ! Hallo world. ! !{Anyone for Tennis?}% !% ! Just another line% !---and another part of the same line ! ! \bye %------------------------------------------------------------------------------- % Not a very sensible input file I know but it's just meant to illustrate the % problem. When run through TeX, I get the following log file (without the % leading `% ' of course): %------------------------------------------------------------------------------- % This is TeX, C Version 3.14t3 (format=plain 93.5.19) 28 MAR 1996 18:31 % **test % (test.tex % % % % % % % % % [1] ) % Output written on test.dvi (1 page, 308 bytes). %------------------------------------------------------------------------------- % I understand all of these except the generated by the input % line `!%'. Why should a comment character generate a ? % % Thanks in anticipation. % % Colin Mason (cgm@sunserver.ssci.liv.ac.uk) ------------------------------ From: Andrew Jones $STAFF Date: Mon, 25 Mar 1996 09:12:44 +0000 Subject: disc set and printer driver for LaTex 2e Please can anyone help me?? I have used Latex in the past, but unfortunately had to remove it. I now possess a Canon BJ10ex printer, and I cannot get the exact driver for it. I've been given several to try, but to no avail. I also understand that I no longer have the latest version of LaTex ( I bought mine in 1990). How is LaTex 2e different? Could anyone please advise me on where I may get a proper printer driver for the BJ10ex, and possibly a disc set for LaTex 2e. I use Wordperfect 6.0 for word-processing. I recall reading somewhere that there is a LaTex converter for WP6. My own machine is not on the Internet, and therefore I cannot download, I would therefore need proper discs. Any help would be greatly appreciated. ------------------------------ End of TeXhax Digest V96 #5 *************************** About TeXhax... Please send contributions to: TeXhax@tex.ac.uk Subscription and unsubscription requests: send a one line mail message to TeXhax-Request@tex.ac.uk containing either subscribe texhax or unsubscribe texhax If you have problems with un/subscribing, please mail owner-texhax@nott.ac.uk To obtain the Frequently Asked Questions (FAQ) lists for TeX, send a message with no subject to fileserv@shsu.edu, consisting of SENDME FAQ For information on the TeX Users Group, please send a message to TUG@TUG.org, or write TeX Users Group, 1850 Union Street, #1637 San Francisco CA 94123 (phone: 1 415 982 8449, fax: 1 415 982 8559) Backnumbers of all the digests are stored in the Comprehensive TeX Archive Network (CTAN) and can be retrieved on the Internet by anonymous ftp. The hosts comprising CTAN include, among others, ftp.dante.de (129.69.1.12) -- Germany ftp.shsu.edu (192.92.115.10) -- USA ftp.tex.ac.uk (128.232.1.87) -- UK Please use your nearest server, to keep network load down. The file /tex-archive/CTAN.sites on each of these hosts gives a list of other sites which maintain full or partial mirrors of the CTAN. Alternatively, finger ctan_us@ftp.shsu.edu for full details. TeXhax Digest back issues are filed below /tex-archive/digests/texhax/ Keyword-In-Context indexes are filed in /tex-archive/digests/indexes/ A Hypermail version of TeXhax is also available on the World-Wide Web at URL http://www.tex.ac.uk/tex-archive/digests/hyper/ \bye From texhax-digest-outgoing-request@nottingham.ac.uk Wed May 29 06:55:10 1996 Flags: 000000000401 Received: from jess.ccc.nottingham.ac.uk (jess.ccc.nottingham.ac.uk [128.243.40.193]) by csc-sun.math.utah.edu (8.7.4/8.7.3) with ESMTP id GAA27954 for ; Wed, 29 May 1996 06:54:41 -0600 (MDT) Message-Id: <199605291254.GAA27954@csc-sun.math.utah.edu> Received: from nottingham.ac.uk by jess.ccc.nottingham.ac.uk id <22537-0@jess.ccc.nottingham.ac.uk>; Wed, 29 May 1996 13:24:21 +0100 From: Majordomo list server To: texhax-digest@nottingham.ac.uk Subject: TeXhax Digest V96 #6 Reply-To: TeXhax@tex.ac.uk Errors-To: owner-texhax-digest@nottingham.ac.uk Precedence: bulk Date: Wed, 29 May 1996 13:24:21 +0100 Sender: owner-texhax-digest@nottingham.ac.uk TeXhax Digest Wednesday, 29 May 1996 Volume 96 : Number 006 (incorporating UKTeX Digest) Today's Topics: "Standard" Font sizes Re: TeXhax Digest V96 #5 TUGboat 17(1) Contents Creating Adobe pdf files from LaTeX excalibur PostScript from other programs [ANNOUNCE] TeX Live CDROM ---------------------------------------------------------------------- From: mclem@medphys.ucl.ac.uk (Matthew Clemence) Date: Mon, 15 Apr 96 15:52:03 BST Subject: "Standard" Font sizes I am maintaining a TeX system here at ucl, and like many others I suspect am allowing the system to generate all the fonts as needed (via MakeTeXpk). All of these end up jumbled together in the texmf/fonts/tmp/pk directory (under either cx or ljfour). I would quite like to shift the common ones into the appropriate sub directory (texmf/fonts/public/cm/ ..) so that tmp can be deleted on a regular basis. Can someone tell me the standard/most common sizes required ? Thanks. - -- ************************************************************************** Dr. Matthew Clemence ___ email mclem@medphys.ucl.ac.uk University College London 11-20 Shropshire House, London, England +44 171 387 9300 x 8448/8264 +44 181 442 1832 Home ************************************************************************** ------------------------------ From: David Carlisle Date: Wed, 17 Apr 96 17:57:14 BST Subject: Re: TeXhax Digest V96 #5 Perhaps the following is a clearer example: \chardef\xxx`\a \ifcat a\xxx \message{yes} \else \message{no} \fi \let\xxx=a \ifcat a\xxx \message{yes} \else \message{no} \fi \bye A \chardef token is not a character token, and so does not have a catcode so as far as \ifcat is concerned it will compare true against any other control sequence, but false against any character. In your case \next is \let to ! ie it has the definition of \futurelet \next \dopling because !% ! is equivalent to !! and \return is a chardef control sequence, so \ifcat\return\noexpand\next finds two control sequences and so returns true. As my example above shows you can get an implicit character token for use in \ifcat by using \let (the \bgroup you used is another such example) but you can not do this for as it is not possible to make any token of catcode 5. input characters of that category always produce tokens of catcode 10 (and sometimes also \par tokens) You can not `see' the end of line by any method at all if you use the normal system, if you make ^^M active then of course you can see a ctacode 13 character token, but you disable the automatic system that TeX uses for converting end of line to white space, so you need arrange that the definition of ^^M does this `by hand'. David ------------------------------ From: Mimi Burbank Date: Tue, 23 Apr 1996 11:16:12 -0500 (EDT) Subject: TUGboat 17(1) Contents TUGboat Volume 17, Number 1 / March 1996 ================================ Addresses 3 Soliciting Bids for TUG'97 4 General Delivery Michel Goossens From the president 5 Barbara Beeton Editorial comments 6 DEK on tour; TUB: the year ahead 6 TUG'95: Questions and answers with Prof. Donald E. Knuth 7 Software & Tools Frank G. Bennett, Jr. Camel: kicking over the bibliographic traces in BibTeX 22 Filip Machi, Jerrold E. Marsden and Wendy G. McKay Corrigendum: Introduction to FasTeX: a system of keyboard shortcuts for the fast keying of TeX (Volume 16(4), pp. 358-363) 28 Fonts Donald E. Knuth Important message regarding CM fonts 29 Darko Zubrinic Croatian fonts 29 Book Reviews J. Vesely Two new books on TeX in the Czech Republic: 34 Petr Olsak, Typograficky system TeX (TeX typesetting system); Jiri Rybicka, LaTeX pro zacatecniky (LaTeX for beginners) Lynne A. Price: Ronald C. Turner, Timothy A. Douglass, and Audrey J. Turner, README.1ST: SGML for Writers and Editors 35 Letters Rama Porrat There's still something missing... 37 Resources Mimi Burbank and Michel Goossens Electronic news from the family 37 Tutorials Keith Reckdahl Using EPS graphics in LaTeXe documents 43 Macros Don Hosek That ol' devil \expandafter 53 J. Hagen and A. F. Otten PPCHTeX: typesetting chemical formulas in TeX 54 LaTeX David Carlisle A LaTeX tour, Part 1: The basic distribution 67 Abstracts Les Cahiers GUTenberg, No. 20 73 News & Announcements Calendar 74 TUG'96 Preliminary schedule 76 Late-Breaking News Mimi Burbank Production notes 78 Future issues 78 TUG Business TUG Bylaws 79 1996 TUG election cancelled 84 TUG Board Members 84 Institutional members 85 Forms TUG membership application 86 Advertisements TeX consulting and production services 87 Index of advertisers 87 ------------------------------ From: "S. Warde" Date: Fri, 26 Apr 1996 12:55:34 +0100 (BST) Subject: Creating Adobe pdf files from LaTeX Looking for help in creating pdf files. 1/ We are currently moving from LaTeX 209 to LaTeX2e. We have found the hyper and hyperref packages in the contrib/supported directory. Though both authors acknowledge each other and say it would be a good idea to merge they don't say when. Until they do I would be grateful for peoples experiences with this packages. 2/ We have obtained the dvihps source from ftp.tex.ac.uk this seems to be quiet old. Before installing it I was wondering if a - there was a more recent version b - anyone had 'ported' it to work with the kpathsea distribution c - knew of any limitations d - knew of other dvips variants for creating pdf files We run LaTeX etc. on SGI's and HP's 3/ We have come across a reference to an archive and mailing list on hypertex hosted at snorri.chem.washington.edu . This machine does not seem to exist. Anyone know if the archive and maillist are hosted elsewhere? Thank you Hugo Korwaser Femsys Limited Tel: +44 (0) 116 2541475 sw25@leiceter.ac.uk ------------------------------ From: sens@sbphy.ucsb.edu (Pierre Sens) Date: Wed, 1 May 1996 11:00:31 +0100 Subject: excalibur Hi Tex users, I would like to use the spell checker Excalibur for documents in French on my Macintosh. I heard about a french dictionnary for Excalibur. Does anybody know where I could download it. Thanks, Pierre Sens Department of Physics UCSB Santa Barbara, Ca 93106 U.S.A Voice 1 (805) 893-8986 Fax 1 (805) 893-2902 email sens@physics.ucsb.edu ------------------------------ From: Russel Winder Date: Tue, 07 May 1996 02:50:36 +0100 Subject: PostScript from other programs I regularly need to include PostScript figures in documents. I find idraw on Unix produces excellent relocatable PostScript descriptions but it is a little simplistic. There are excellent command language mechanisms for creating PostScript tricks using LaTeX but I really want to make use of (essentially) WYSIWYG programs like idraw, FrameMaker and Word to create images for inclusion in LaTeX2e documents. Furthermore I often have the requirement to use other people's PostScript in my LaTeX documents and have no control over the programs used to create the PostScript. In the past I have been able to use PostScript produced by various Macintosh programs (though judicious editing out of font information is usually required) and also PostScript produced by the PostScript driver under Windows3.1. At the pther extreme, FrameMaker has always been a pain in the !@#$. The PostScript produced by this program is totally un-relocatable. It assumes total control of the page and I have yet to discover the necessary edits to remove it's dictatorial efforts and hence make the PostScript relocatable. Moreover Microsoft appear to have gone down this same road and this is my real problem. Word7 with the Apple LaaserWriter plus driver (under Windows95) now produces code that it not relocatable. Indeed it has this penchant for trying to enquire of the amount of virtual memory on a regular basis which makes the PostScript file of a document un-previewable with ghostscript. I have found the trick for removing this VM quesry problem but have not found the trick for removing the manipulations that bind the coordinate system to the page in it's dictatorial manner. Is there a pool of experience on (or even tools to help forcing Word7 and FrameMaker PostScript into a relocatable and hence usable form? Thanks for any guidance and/or pointers. Russel. ======================================================================= Dr Russel Winder Reader in Software Engineering Editor-in-Chief, Object Oriented Systems Information Systems Research Group Department of Computer Science Phone: +44 (0)171 380 7293 University College London Fax: +44 (0)171 387 1397 Gower Street EMail: R.Winder@cs.ucl.ac.uk London WC1E 6BT UK URL: http://www.cs.ucl.ac.uk/staff/R.Winder/ ======================================================================= ------------------------------ From: Sebastian Rahtz Date: Wed, 29 May 1996 13:19:48 +0100 Subject: [ANNOUNCE] TeX Live CDROM I am very glad to announce the launch today in Paris of TeX Live, a new CD published by the TeX Users Group, the UK TeX Users Group and GUIenberg (French TeX Users), with help from NTG (Dutch TeX Users) and many individuals from other groups. TeX Live's 649 megabytes contains: - a ready to run Unix TeX setup, Thomas Esser's teTeX (based on Karl Berry's Web2c). It has binaries for: Linux on Intel and m68k platforms; Irix 5.2, 5.3 on MIPS (SGI Indy/Indigo) SunOS 4.1.3 on Sun Solaris 2.3, 2.4, 2.5 on SPARC HPUX 9.01, 10.01 for HP workstations Digital Unix (OSF/1) 2.0 and 3.2 for DEC Alpha machines FreeBSD and NetBSD on Intel platforms Ultrix 4.3, for DEC Decstation machines AIX 3.2, 4.1.1, for IBM RS6000 machines NeXTStep on Intel platforms - a very large support tree of macros, fonts and documentation arranged according to the TeX Directory Structure layout - the GUTenberg Mac, DOS and Windows distributions (archived) You can use the TeX system by running directly from the CD, installing on your hard disk, or by adding packages to your existing system. This is not a dump of CTAN full of compressed archives. This is a *working* system. To make it useable under Unix, it uses the Rock Ridge extensions to the ISO9660 file system. Ordinary systems can still read it, but will not see the long file names or symbolic links. If you are on any flavour of Unix, BUY this CD! There is no complicated compilation or moving of installed files around, it will just *WORK*. More details, and ordering information, can be found at http://www.tug.org/texlive.html. If you dont have WWW access, mail me or tug@tug.org for details. Cost is around $20 for members of any TeX users group, or $40 for others (the prices vary, depending on postage). All profits from sales go back to fund new versions of the CD, and to TeX-related development projects. Sebastian Rahtz Secretary, TeX Users Group ------------------------------ End of TeXhax Digest V96 #6 *************************** About TeXhax... Please send contributions to: TeXhax@tex.ac.uk Subscription and unsubscription requests: send a one line mail message to TeXhax-Request@tex.ac.uk containing either subscribe texhax or unsubscribe texhax If you have problems with un/subscribing, please mail owner-texhax@nott.ac.uk To obtain the Frequently Asked Questions (FAQ) lists for TeX, send a message with no subject to fileserv@shsu.edu, consisting of SENDME FAQ For information on the TeX Users Group, please send a message to TUG@TUG.org, or write TeX Users Group, 1850 Union Street, #1637 San Francisco CA 94123 (phone: 1 415 982 8449, fax: 1 415 982 8559) Backnumbers of all the digests are stored in the Comprehensive TeX Archive Network (CTAN) and can be retrieved on the Internet by anonymous ftp. The hosts comprising CTAN include, among others, ftp.dante.de (129.69.1.12) -- Germany ftp.shsu.edu (192.92.115.10) -- USA ftp.tex.ac.uk (128.232.1.87) -- UK Please use your nearest server, to keep network load down. The file /tex-archive/CTAN.sites on each of these hosts gives a list of other sites which maintain full or partial mirrors of the CTAN. Alternatively, finger ctan_us@ftp.shsu.edu for full details. TeXhax Digest back issues are filed below /tex-archive/digests/texhax/ Keyword-In-Context indexes are filed in /tex-archive/digests/indexes/ A Hypermail version of TeXhax is also available on the World-Wide Web at URL http://www.tex.ac.uk/tex-archive/digests/hyper/ \bye From texhax-digest-outgoing-request@nottingham.ac.uk Wed May 29 10:05:35 1996 Flags: 000000000001 Received: from jess.ccc.nottingham.ac.uk (jess.ccc.nottingham.ac.uk [128.243.40.193]) by csc-sun.math.utah.edu (8.7.4/8.7.3) with ESMTP id KAA03130 for ; Wed, 29 May 1996 10:05:23 -0600 (MDT) Message-Id: <199605291605.KAA03130@csc-sun.math.utah.edu> Received: from nottingham.ac.uk by jess.ccc.nottingham.ac.uk id <25657-0@jess.ccc.nottingham.ac.uk>; Wed, 29 May 1996 16:14:30 +0100 From: Majordomo list server To: texhax-digest@nottingham.ac.uk Subject: TeXhax Digest V96 #7 Reply-To: TeXhax@tex.ac.uk Errors-To: owner-texhax-digest@nottingham.ac.uk Precedence: bulk Date: Wed, 29 May 1996 16:14:30 +0100 Sender: owner-texhax-digest@nottingham.ac.uk TeXhax Digest Wednesday, 29 May 1996 Volume 96 : Number 007 (incorporating UKTeX Digest) Today's Topics: Re: TeXhax Digest V96 #6 Re: TeXhax Digest V96 #6 Redefining maximum number of strings, fonts, etc... margins BibTeX/LaTeX2e question BibTeX/LaTeX2e point -- PS Location of index package (by David Jones)??? \def with catcode 1 in parameter text? Re: TeXhax Digest V96 #6 Re: TeXhax Digest V96 #6 ---------------------------------------------------------------------- From: Sebastian Rahtz Date: Wed, 29 May 1996 14:06:40 +0100 Subject: Re: TeXhax Digest V96 #6 > From: "S. Warde" < > Date: Fri, 26 Apr 1996 12:55:34 +0100 (BST) > Subject: Creating Adobe pdf files from LaTeX > We are currently moving from LaTeX 209 to LaTeX2e. We have > found the hyper and hyperref packages in the contrib/supported > directory. > Though both authors acknowledge each other and say it would be > a good idea to merge they don't say when. Until they do I would tell us if its necessary, and I expect we'll do something about it. > We have obtained the dvihps source from ftp.tex.ac.uk this seems > to be quiet old. Before installing it I was wondering if > a - there was a more recent version the author is working on it > b - anyone had 'ported' it to work with the kpathsea > distribution yes, its moderately trivial to do yourself. but i'll be in the next release of dvipsk > c - knew of any limitations yes, you cant vary the height of the anchor box... > d - knew of other dvips variants for creating pdf files use the nativepdf option of hyperref, and you dont need dvihps at all sebastian rahtz ------------------------------ From: Sebastian Rahtz Date: Wed, 29 May 1996 14:09:02 +0100 Subject: Re: TeXhax Digest V96 #6 > From: mclem@medphys.ucl.ac.uk (Matthew Clemence) > From: Russel Winder > Date: Tue, 07 May 1996 02:50:36 +0100 > Subject: PostScript from other programs > Is there a pool of experience on (or even tools to help forcing Word7 > and FrameMaker PostScript into a relocatable and hence usable form? my generic solution is Acrobat Distiller. its an excellent (obviously) PostScript interpreter. Use that to make a PDF (ie clean!) version of the file, then use Acrobat Exchange to write out a new .ps version. i havent tried this per se with Frame files, but the principal should be sound. sebastian ------------------------------ From: mse@sdr.utias.utoronto.ca (Manfred D. M. Sever) Date: Mon, 13 May 1996 13:51:46 -0400 Subject: Redefining maximum number of strings, fonts, etc... Hello All! I would like to be able to increase the maximum number ``strings'' available to TeX for processing documents. It may also be usefull to increase the upper limit on the number of fonts allowed. I think this should be possible to do since Goossens, Mittelbach and Samarin discuss this on page xi of their book. I believe that increasing these limits also involves recompiling TeX. However, I can't find the file that these limits are set in. How do I proceed? Thanks in advance, Manfred. ------------------------------ From: Mr Marcilo Alves Date: Thu, 16 May 1996 10:11:57 +0100 (BST) Subject: margins Hi, That's Marcilio from Liverpool. I am using book class in Latex2e with the option [a4paper,twoside]. Yet, my output shows different margins; that is there is a horizontal and vertical shift of one side of the output in relation to the other side. Is this a problem with the book class or with the printer. The printer is in the computer centre and I do not know how to eventually set margins, etc... Could you help me? - - --------------------------------- I am using fancyheadings to set the head of the page. It works fine but it seems that the commands \listoffigures \listoftables \tableofcontents uses its own formatting for the heads. How can I set my own head in conjunction with the above commands? - - --------------------------------- I do appreciate your help in advance! Thank you marcilio ------------------------------ From: Russel Winder Date: Thu, 16 May 1996 19:31:37 +0100 Subject: BibTeX/LaTeX2e question I am wondering whether there is a solution to this or whether it is "just one of those things you have to hack around". The background: Using the Harvard family of citation styles with BibTeX and LaTeX2e. Two references are different papers in the same tome. Cross-referencing is used to avoid duplication of information. Both references are used in the same document. The tome itself (item C) has no author or editor. @InCollection{A, ... crossref = "C" } @InCollection{B, ... crossref = "C" } @Book{C, ... } The problem: Because the tome is cited in two difference references, three entries get written to the bbl and auxiliary files. Because the tome has no author or editor, the title is used as the citation mark (both short and long form) -- and here is the problem -- this is emphasized, i.e. there is a \emph in the parameters of the citation entries in the .aux file. When the aux file is read in, the error: ! Incomplete \iffalse; all text was ignored after line 72. \fi occurs. There are three hack solutions: 1. Manual edit the .aux file -- totally unacceptable. 2. Do not use crossref but suffer the replication of information -- unacceptable but... 3. Force BibTeX not to generate the third reference item but to replicate crossreference information in all entries -- I haven't investigated this and therefore do not know how to do it. Have others seen this and found other solutions? Russel. ======================================================================= Dr Russel Winder Reader in Software Engineering Editor-in-Chief, Object Oriented Systems Information Systems Research Group Department of Computer Science Phone: +44 (0)171 380 7293 University College London Fax: +44 (0)171 387 1397 Gower Street EMail: R.Winder@cs.ucl.ac.uk London WC1E 6BT UK URL: http://www.cs.ucl.ac.uk/staff/R.Winder/ ======================================================================= ------------------------------ From: Russel Winder Date: Thu, 16 May 1996 19:48:19 +0100 Subject: BibTeX/LaTeX2e point -- PS I forgot to note the obvious point about using the key field in the bibliography item, I suppose that should be solution 0. My point was: Has anyone got an answer to the problem of \emph appearing in the .aux file, even if accidently? ------------------------------ From: coleman@trillium.phys.uregina.ca (Robert Coleman) Date: Fri, 17 May 1996 15:28:01 -0600 (CST) Subject: Location of index package (by David Jones)??? On page 367 of the LaTeX Companion, the index package written by David Jones is discussed. I have, however, not been able to find it on CTAN. From where can this package be ftped? Thanks for the help. A direct reply would be appreciated. Robert - -- Robert Alan Coleman Department of Physics University of Regina Regina, Saskatchewan Canada S4S 0A2 Tel: (306) 585-4260 Fax: (306) 585-4894 email: coleman@cas.uregina.ca ------------------------------ From: Matteo Frigo Date: Mon, 27 May 1996 11:41:45 -0400 Subject: \def with catcode 1 in parameter text? Abstract: I want to put a token with catcode 1 in the parameter text of a macro, and I don't know how. Problem: I wrote a macro \largeenough which takes one argument, and formats it in the largest possible font so that the argument fits in one page (I use it for slides). The macro works by trial and error, and some other hackery. Now, I want to define a LaTeX environment \begin{largeenough} text \end{largeenough} to accomplish the same effect --- since the environment seems more elegant to me. The problem is: how do I capture all text preceding the \end{largeenough} ? I need it for the trial-and-error procedure. So far, I use a simple-minded loop to accomplish this effect, but I wonder if there is a better way. Essentially, I would like to define a macro \def\foo#1\end{largeenough}{...} where I have a { *with catcode 1* in the parameter text. Changing the catcode doesn't work, because I want to keep the original catcode of { in the normal text (inside the environment). Any idea? Thanks, Matteo ------------------------------ From: Michael Doob Date: Wed, 29 May 1996 08:39:09 -0500 (CDT) Subject: Re: TeXhax Digest V96 #6 > From: mclem@medphys.ucl.ac.uk (Matthew Clemence) > Date: Mon, 15 Apr 96 15:52:03 BST > Subject: "Standard" Font sizes > > I am maintaining a TeX system here at ucl, and like many others I suspect > am allowing the system to generate all the fonts as needed (via MakeTeXpk). > All of these end up jumbled together in the texmf/fonts/tmp/pk directory > (under either cx or ljfour). I would quite like to shift the common ones > into the appropriate sub directory (texmf/fonts/public/cm/ ..) so that > tmp can be deleted on a regular basis. Can someone tell me the > standard/most common sizes required ? You could run a cron job that deletes pk files that haven't been accessed recently. What's left over will be (by definition) the popular ones, and they could be moved out to a more permanent location periodically. Cheers, Michael ------------------------------ From: David Carlisle Date: Wed, 29 May 1996 15:32:12 +0100 Subject: Re: TeXhax Digest V96 #6 > Date: Fri, 26 Apr 1996 12:55:34 +0100 (BST) You'd get a quicker information turn around by asking on comp.text.tex newsgroup (or the equivalent info-tex mailing list). > b - anyone had 'ported' it [dvihps] to work with the kpathsea > distribution I'm not sure, but if you use the [nativepdf] option to the hyperref package you can use a standard dvips(k). The pdfmark operators are written straight to the PS file via literal postscript specials. > on hypertex hosted at snorri.chem.washington.edu . This machine > does not seem to exist the mailing list is now reachable via .... This mailing list will ensure that you are notified of new releases and other major developments to HyperTeX, the hypertext extensions to TeX. The list address is: hypertex-announce@aps.org This list is now being managed by majordomo, so subscription requests and other information can be obtained by e-mail to majordomo@aps.org Archives of the announcements and an ftp area for the software should be announced shortly. For other information, check out the Web page at Los Alamos: http://xxx.lanl.gov/hypertex/ Please let the list owner (apsmith@aps.org) know of any problems. ------------------------------ End of TeXhax Digest V96 #7 *************************** About TeXhax... Please send contributions to: TeXhax@tex.ac.uk Subscription and unsubscription requests: send a one line mail message to TeXhax-Request@tex.ac.uk containing either subscribe texhax or unsubscribe texhax If you have problems with un/subscribing, please mail owner-texhax@nott.ac.uk To obtain the Frequently Asked Questions (FAQ) lists for TeX, send a message with no subject to fileserv@shsu.edu, consisting of SENDME FAQ For information on the TeX Users Group, please send a message to TUG@TUG.org, or write TeX Users Group, 1850 Union Street, #1637 San Francisco CA 94123 (phone: 1 415 982 8449, fax: 1 415 982 8559) Backnumbers of all the digests are stored in the Comprehensive TeX Archive Network (CTAN) and can be retrieved on the Internet by anonymous ftp. The hosts comprising CTAN include, among others, ftp.dante.de (129.69.1.12) -- Germany ftp.shsu.edu (192.92.115.10) -- USA ftp.tex.ac.uk (128.232.1.87) -- UK Please use your nearest server, to keep network load down. The file /tex-archive/CTAN.sites on each of these hosts gives a list of other sites which maintain full or partial mirrors of the CTAN. Alternatively, finger ctan_us@ftp.shsu.edu for full details. TeXhax Digest back issues are filed below /tex-archive/digests/texhax/ Keyword-In-Context indexes are filed in /tex-archive/digests/indexes/ A Hypermail version of TeXhax is also available on the World-Wide Web at URL http://www.tex.ac.uk/tex-archive/digests/hyper/ \bye From texhax-digest-outgoing-request@nottingham.ac.uk Tue Jun 18 03:37:11 1996 Flags: 000000000001 Received: from jess.ccc.nottingham.ac.uk (jess.ccc.nottingham.ac.uk [128.243.40.193]) by csc-sun.math.utah.edu (8.7.4/8.7.3) with ESMTP id DAA09569 for ; Tue, 18 Jun 1996 03:37:07 -0600 (MDT) Message-Id: <199606180937.DAA09569@csc-sun.math.utah.edu> Received: from nottingham.ac.uk by jess.ccc.nottingham.ac.uk id <14408-0@jess.ccc.nottingham.ac.uk>; Tue, 18 Jun 1996 10:17:25 +0100 From: Majordomo list server To: texhax-digest@nottingham.ac.uk Subject: TeXhax Digest V96 #8 Reply-To: TeXhax@tex.ac.uk Errors-To: owner-texhax-digest@nottingham.ac.uk Precedence: bulk Date: Tue, 18 Jun 1996 10:17:25 +0100 Sender: owner-texhax-digest@nottingham.ac.uk TeXhax Digest Tuesday, 18 June 1996 Volume 96 : Number 008 (incorporating UKTeX Digest) Today's Topics: Re: TeXhax Digest V96 #7 Re: TeXhax Digest V96 #7 Re: TeXhax Digest V96 #7 Re: Location of David Jones' index package (TeXhax V96 #7) Re: excalibur TeXhax Digest V96 #6 Writing long formulas Fonts in Latex2.09 [none] EMTeX no longer available from ftp.shsu.edu ---------------------------------------------------------------------- From: Sebastian Rahtz Date: Thu, 30 May 1996 08:22:44 +0100 Subject: Re: TeXhax Digest V96 #7 > I would like to be able to increase the > maximum number ``strings'' available to > TeX for processing documents. It may also > be usefull to increase the upper limit > on the number of fonts allowed. yes, you can do it. but its not trivial. current releases of TeX for DOS (emtex) and Mac (OzTeX) allow you to do it dynamically), while Textures and Y&Y TeX claim to just give you unlimited memory (I only say claim because i havent tried it myself). If you have Unix TeX, lets assume you have the web2c stuff. in that case, find the directory web2c/tex and the file ctex.ch. thats the "change file", and near the top you find a set of constants which are set to complicated values. thats where you play, and thats as far as I am going to give a recipe! you have to understand how change files work (read the WEB documentation) and what the values do. NOTE that some values are already at their upper limit sebastian rahtz ------------------------------ From: David Carlisle Date: Wed, 29 May 1996 17:03:05 +0100 Subject: Re: TeXhax Digest V96 #7 > Abstract: I want to put a token with catcode 1 in the parameter text > of a macro, and I don't know how. You can't. > Essentially, I would like to define a macro > \def\foo#1\end{largeenough}{...} you have to go \def\foo#1\end#2{... and then check that #2 is `largeenough' and if not search for the next \end. It's a bit of a pain, but that's life. There are examples of this in the ams alignment environments, and tabularx and load of other places. David ------------------------------ From: Matteo Frigo Date: Wed, 29 May 1996 12:07:58 -0400 Subject: Re: TeXhax Digest V96 #7 > \def\foo#1\end#2{... > and then check that #2 is `largeenough' and if not search for the next > \end. It's a bit of a pain, but that's life. There are examples of this > in the ams alignment environments, and tabularx and load of other > places. > > David > Thanks for your answer. In fact, this is what I am doing now. I am glad to hear that there is no better way to do the same thing. Cheers Matteo ------------------------------ From: Robin Fairbairns Date: Wed, 29 May 1996 17:32:15 +0100 Subject: Re: Location of David Jones' index package (TeXhax V96 #7) Robert Coleman writes: >On page 367 of the LaTeX Companion, the index >package written by David Jones is discussed. >I have, however, not been able to find it on >CTAN. [...] I maintain a file (on CTAN, of course) info/companion.ctan; this aims to list all packages mentioned in the Companion, and to give their location on CTAN. In this instance, it says index macros/latex209/contrib/misc/index.doc As you will note, this is a 2.09 package. A 2e one is in development, and I have a copy of a beta-version. Sadly, I failed to note which MIT machine I got it from, and it's pretty old. Perhaps David Jones himself will respond... (Or even submit a new version for the archive?) - -- Robin (Campaign for Real Radio 3) Fairbairns rf@cl.cam.ac.uk U of Cambridge Computer Lab, Pembroke St, Cambridge CB2 3QG, UK Home page: http://www.cl.cam.ac.uk/users/rf/robin.html ------------------------------ From: Alun.J.Carr@ucd.ie (Dr Alun J. Carr) Date: Thu, 30 May 1996 10:55:33 +0100 Subject: Re: excalibur On Wed, 1 May 1996 11:00:31 +0100 sens@sbphy.ucsb.edu (Pierre Sens) wrote: >Hi Tex users, > >I would like to use the spell checker Excalibur for documents in French on >my Macintosh. I heard about a french dictionnary for Excalibur. Does >anybody know where I could download it. Alun - -- Dr Alun J. CARR Phone: +353-1-7061989 Mechanical Engineering Dept. +353-1-2693244 x1989 University College Dublin Fax: +353-1-2830534 Belfield E-mail: Dublin 4 WWW: Ireland ------------------------------ From: bkph@ai.mit.edu (Berthold K.P. Horn) Date: Fri, 31 May 1996 08:34:23 -0400 Subject: TeXhax Digest V96 #6 > From: Russel Winder > Date: Tue, 07 May 1996 02:50:36 +0100 > Subject: PostScript from other programs > > I regularly need to include PostScript figures in documents. I find > idraw on Unix produces excellent relocatable PostScript descriptions > but it is a little simplistic. There are excellent command language > mechanisms for creating PostScript tricks using LaTeX but I really > want to make use of (essentially) WYSIWYG programs like idraw, > FrameMaker and Word to create images for inclusion in LaTeX2e > documents. Furthermore I often have the requirement to use other > people's PostScript in my LaTeX documents and have no control over the > programs used to create the PostScript. > > In the past I have been able to use PostScript produced by various > Macintosh programs (though judicious editing out of font information > is usually required) and also PostScript produced by the PostScript > driver under Windows3.1. > > At the pther extreme, FrameMaker has always been a pain in the !@#$. > The PostScript produced by this program is totally un-relocatable. It > assumes total control of the page and I have yet to discover the > necessary edits to remove it's dictatorial efforts and hence make the > PostScript relocatable. > > Moreover Microsoft appear to have gone down this same road and this is > my real problem. Word7 with the Apple LaaserWriter plus driver (under > Windows95) now produces code that it not relocatable. Indeed it has > this penchant for trying to enquire of the amount of virtual memory on > a regular basis which makes the PostScript file of a document > un-previewable with ghostscript. I have found the trick for removing > this VM quesry problem but have not found the trick for removing the > manipulations that bind the coordinate system to the page in it's > dictatorial manner. > > Is there a pool of experience on (or even tools to help forcing Word7 > and FrameMaker PostScript into a relocatable and hence usable form? > > Thanks for any guidance and/or pointers. > > Russel. > > ======================================================================= > > Dr Russel Winder > > Reader in Software Engineering > Editor-in-Chief, Object Oriented Systems > > Information Systems Research Group > Department of Computer Science Phone: +44 (0)171 380 7293 > University College London Fax: +44 (0)171 387 1397 > Gower Street EMail: R.Winder@cs.ucl.ac.uk > London WC1E 6BT > UK URL: http://www.cs.ucl.ac.uk/staff/R.Winder/ > > ======================================================================= You must make sure the applications export in EPS format. If you use raw PS, you are asking for trouble, because PS drivers are allowed to use arbitrary PS code including the forbidden-in-EPS operators that are troubling you. Most printer drivers have a check box this produce EPS output. If you can't use EPS because the output is multi-page then you are not using EPS, since that by definition is one page only. Berthold. ------------------------------ From: Dominique de Waleffe Date: Wed, 5 Jun 96 15:42:51 +0200 Subject: Writing long formulas In Digital SRC report 119, Leslie Lamport presents a nice way to format long mathematical formulas so that reading is a lot easier. Does anyone know a Latex package which implements those conventions? I'd be very interested. D. - -- Dominique de Waleffe Mission Critical, Wijnegemhofstraat 199, B-3071 Erps-Kwerps (Belgium) Phone: +32 2 759 95 60 Fax: +32 2 759 27 60 email: ddw@acm.org, ddw@miscrit.be PGP key fingerprint: F9 CC 23 74 44 62 7C F3 8C 12 DF 71 BB 60 54 98 ------------------------------ From: "Klaus M. Wendel" Date: Tue, 11 Jun 1996 11:56:13 +0200 (MSZ) Subject: Fonts in Latex2.09 Help! I am using LaTex Version 2.09 and i have to print a document in HELVETICA. Is there a (easy) way to do this? Thanks! - --------------------------------------------------- Klaus M. Wendel always :-) E-Mail: a9304890@unet.univie.ac.at WWW: http://www.wu-wien.ac.at/usr/h93/h9304890/ - --------------------------------------------------- ------------------------------ From: David Simpson Date: Mon, 17 Jun 1996 09:53:46 +0100 Subject: [none] Does anybody know how to convert LaTeX(2e) files to, dare I say, Word or Wordperfect documents? Apologies to all tex fans for asking this, but unfortunately I can't persuade all my PC-based colleagues to switch to LateX, but I need to send them text for reports. Thanks, Dave Simpson (david.simpson@dnmi.no) ------------------------------ From: Rainer Schoepf Date: Tue, 18 Jun 1996 11:14:13 +0200 (MET DST) Subject: EMTeX no longer available from ftp.shsu.edu As ftp.shsu.edu is not properly managed, and the emTeX files there are partially out of date, I removed emTeX at teh request of Eberhard Mattes from ftp.shsu.edu. We are apologize for the inconvenience, but the CTAN team doesn't have the resources to manage a site from across the atlantic. For the CTAN team - -- Rainer Schvpf Zentrum f|r Datenverarbeitung A point of view can be a dangerous der Universitdt Mainz luxury when substituted for insight Anselm-Franz-von-Bentzel-Weg 12 and understanding. D-55099 Mainz Germany Herbert Marshall McLuhan: The Gutenberg Galaxy ------------------------------ End of TeXhax Digest V96 #8 *************************** About TeXhax... Please send contributions to: TeXhax@tex.ac.uk Subscription and unsubscription requests: send a one line mail message to TeXhax-Request@tex.ac.uk containing either subscribe texhax or unsubscribe texhax If you have problems with un/subscribing, please mail owner-texhax@nott.ac.uk To obtain the Frequently Asked Questions (FAQ) lists for TeX, send a message with no subject to fileserv@shsu.edu, consisting of SENDME FAQ For information on the TeX Users Group, please send a message to TUG@TUG.org, or write TeX Users Group, 1850 Union Street, #1637 San Francisco CA 94123 (phone: 1 415 982 8449, fax: 1 415 982 8559) Backnumbers of all the digests are stored in the Comprehensive TeX Archive Network (CTAN) and can be retrieved on the Internet by anonymous ftp. The hosts comprising CTAN include, among others, ftp.dante.de (129.69.1.12) -- Germany ftp.shsu.edu (192.92.115.10) -- USA ftp.tex.ac.uk (128.232.1.87) -- UK Please use your nearest server, to keep network load down. The file /tex-archive/CTAN.sites on each of these hosts gives a list of other sites which maintain full or partial mirrors of the CTAN. Alternatively, finger ctan_us@ftp.shsu.edu for full details. TeXhax Digest back issues are filed below /tex-archive/digests/texhax/ Keyword-In-Context indexes are filed in /tex-archive/digests/indexes/ A Hypermail version of TeXhax is also available on the World-Wide Web at URL http://www.tex.ac.uk/tex-archive/digests/hyper/ \bye From texhax-digest-outgoing-request@nottingham.ac.uk Tue Jun 18 11:28:29 1996 Flags: 000000000001 Received: from jess.ccc.nottingham.ac.uk (jess.ccc.nottingham.ac.uk [128.243.40.193]) by csc-sun.math.utah.edu (8.7.4/8.7.3) with ESMTP id LAA28401 for ; Tue, 18 Jun 1996 11:28:09 -0600 (MDT) Message-Id: <199606181728.LAA28401@csc-sun.math.utah.edu> Received: from nottingham.ac.uk by jess.ccc.nottingham.ac.uk id <00363-0@jess.ccc.nottingham.ac.uk>; Tue, 18 Jun 1996 18:09:36 +0100 From: Majordomo list server To: texhax-digest@nottingham.ac.uk Subject: TeXhax Digest V96 #9 Reply-To: TeXhax@tex.ac.uk Errors-To: owner-texhax-digest@nottingham.ac.uk Precedence: bulk Date: Tue, 18 Jun 1996 18:09:36 +0100 Sender: owner-texhax-digest@nottingham.ac.uk TeXhax Digest Tuesday, 18 June 1996 Volume 96 : Number 009 (incorporating UKTeX Digest) Today's Topics: pagebreaks, &c in indexes (resubmitted) Re: TeXhax Digest V96 #8 ---------------------------------------------------------------------- From: David Handscomb Date: Tue, 18 Jun 96 10:45:08 BST Subject: pagebreaks, &c in indexes (resubmitted) As nobody responded when I first put this question, may I have another go? Surely there is a `makeindex' guru somewhere out there? - ----- Begin Included Message ----- >From dch Wed Oct 4 17:41:49 1995 To: TeXhax@tex.ac.uk Subject: pagebreaks, &c in indexes Content-Length: 869 X-Lines: 41 a) Is there any way of ensuring that an index generated by makeindex is not broken between an item and its following subitem? - e.g.: network 12 news ========= pagebreak ============ bad 13 good 14 next 15 I guess (see Companion p.359) that it could possibly be done via the keyword item_x1 in the *.ist file, if at all. If so, then how? \nopagebreak in the *.ind file seems to have no effect. b) Furthermore, is there any way of generating the following? My guess is `no': news bad 13 good 14 \emph{see also} gnus David Handscomb Numerical Analysis Group Oxford University Computing Laboratory Wolfson Building Parks Road Oxford OX1 3QD ENGLAND tel. (national) 01865 273891 (international) +44 1865 273891 FAX (national) 01865 273839 (international) +44 1865 273839 E-mail dch@comlab.ox.ac.uk / na.handscomb@na-net.ornl.gov - ----- End Included Message ----- ------------------------------ From: "e-Floyd B. Hanson, UIC 312-413-2142" Date: Tue, 18 Jun 1996 12:08:18 CDT Subject: Re: TeXhax Digest V96 #8 > TeXhax Digest Tuesday, 18 June 1996 Volume 96 : Number 008 > > (incorporating UKTeX Digest) > > Today's Topics: > > Re: TeXhax Digest V96 #7 > Re: TeXhax Digest V96 #7 > Re: TeXhax Digest V96 #7 > Re: Location of David Jones' index package (TeXhax V96 #7) > Re: excalibur > TeXhax Digest V96 #6 > Writing long formulas > Fonts in Latex2.09 > [none] > EMTeX no longer available from ftp.shsu.edu > > ---------------------------------------------------------------------- > > From: Sebastian Rahtz > Date: Thu, 30 May 1996 08:22:44 +0100 > Subject: Re: TeXhax Digest V96 #7 > > > I would like to be able to increase the > > maximum number ``strings'' available to > > TeX for processing documents. It may also > > be usefull to increase the upper limit > > on the number of fonts allowed. > yes, you can do it. but its not trivial. current releases of TeX for > DOS (emtex) and Mac (OzTeX) allow you to do it dynamically), while > Textures and Y&Y TeX claim to just give you unlimited memory (I only say > claim because i havent tried it myself). If you have Unix TeX, lets > assume you have the web2c stuff. in that case, find the directory > web2c/tex and the file ctex.ch. thats the "change file", and near the > top you find a set of constants which are set to complicated > values. thats where you play, and thats as far as I am going to give a > recipe! you have to understand how change files work (read the WEB > documentation) and what the values do. NOTE that some values are > already at their upper limit > > sebastian rahtz > > > ------------------------------ > > From: David Carlisle > Date: Wed, 29 May 1996 17:03:05 +0100 > Subject: Re: TeXhax Digest V96 #7 > >> Abstract: I want to put a token with catcode 1 in the parameter text >> of a macro, and I don't know how. > > You can't. > >> Essentially, I would like to define a macro > >> \def\foo#1\end{largeenough}{...} > > you have to go > > \def\foo#1\end#2{... > and then check that #2 is `largeenough' and if not search for the next > \end. It's a bit of a pain, but that's life. There are examples of this > in the ams alignment environments, and tabularx and load of other > places. > > David > > > > ------------------------------ > > From: Matteo Frigo > Date: Wed, 29 May 1996 12:07:58 -0400 > Subject: Re: TeXhax Digest V96 #7 > >> \def\foo#1\end#2{... >> and then check that #2 is `largeenough' and if not search for the next >> \end. It's a bit of a pain, but that's life. There are examples of this >> in the ams alignment environments, and tabularx and load of other >> places. >> >> David >> > > Thanks for your answer. In fact, this is what I am doing now. I am > glad to hear that there is no better way to do the same thing. > > Cheers > Matteo > > > ------------------------------ > > From: Robin Fairbairns > Date: Wed, 29 May 1996 17:32:15 +0100 > Subject: Re: Location of David Jones' index package (TeXhax V96 #7) > > Robert Coleman writes: >>On page 367 of the LaTeX Companion, the index >>package written by David Jones is discussed. >>I have, however, not been able to find it on >>CTAN. [...] > > I maintain a file (on CTAN, of course) info/companion.ctan; this aims > to list all packages mentioned in the Companion, and to give their > location on CTAN. > > In this instance, it says > > index > macros/latex209/contrib/misc/index.doc > > As you will note, this is a 2.09 package. A 2e one is in development, > and I have a copy of a beta-version. Sadly, I failed to note which > MIT machine I got it from, and it's pretty old. Perhaps David Jones > himself will respond... (Or even submit a new version for the > archive?) > - -- > Robin (Campaign for Real Radio 3) Fairbairns rf@cl.cam.ac.uk > U of Cambridge Computer Lab, Pembroke St, Cambridge CB2 3QG, UK > Home page: http://www.cl.cam.ac.uk/users/rf/robin.html > > > ------------------------------ > > From: Alun.J.Carr@ucd.ie (Dr Alun J. Carr) > Date: Thu, 30 May 1996 10:55:33 +0100 > Subject: Re: excalibur > > On Wed, 1 May 1996 11:00:31 +0100 sens@sbphy.ucsb.edu (Pierre Sens) wrote: >>Hi Tex users, >> >>I would like to use the spell checker Excalibur for documents in French on >>my Macintosh. I heard about a french dictionnary for Excalibur. Does >>anybody know where I could download it. > > x> > > Alun > > > - -- > Dr Alun J. CARR Phone: +353-1-7061989 > Mechanical Engineering Dept. +353-1-2693244 x1989 > University College Dublin Fax: +353-1-2830534 > Belfield E-mail: > Dublin 4 WWW: > Ireland > > > ------------------------------ > > From: bkph@ai.mit.edu (Berthold K.P. Horn) > Date: Fri, 31 May 1996 08:34:23 -0400 > Subject: TeXhax Digest V96 #6 > >> From: Russel Winder >> Date: Tue, 07 May 1996 02:50:36 +0100 >> Subject: PostScript from other programs >> >> I regularly need to include PostScript figures in documents. I find >> idraw on Unix produces excellent relocatable PostScript descriptions >> but it is a little simplistic. There are excellent command language >> mechanisms for creating PostScript tricks using LaTeX but I really >> want to make use of (essentially) WYSIWYG programs like idraw, >> FrameMaker and Word to create images for inclusion in LaTeX2e >> documents. Furthermore I often have the requirement to use other >> people's PostScript in my LaTeX documents and have no control over the >> programs used to create the PostScript. >> >> In the past I have been able to use PostScript produced by various >> Macintosh programs (though judicious editing out of font information >> is usually required) and also PostScript produced by the PostScript >> driver under Windows3.1. >> >> At the pther extreme, FrameMaker has always been a pain in the !@#$. >> The PostScript produced by this program is totally un-relocatable. It >> assumes total control of the page and I have yet to discover the >> necessary edits to remove it's dictatorial efforts and hence make the >> PostScript relocatable. >> >> Moreover Microsoft appear to have gone down this same road and this is >> my real problem. Word7 with the Apple LaaserWriter plus driver (under >> Windows95) now produces code that it not relocatable. Indeed it has >> this penchant for trying to enquire of the amount of virtual memory on >> a regular basis which makes the PostScript file of a document >> un-previewable with ghostscript. I have found the trick for removing >> this VM quesry problem but have not found the trick for removing the >> manipulations that bind the coordinate system to the page in it's >> dictatorial manner. >> >> Is there a pool of experience on (or even tools to help forcing Word7 >> and FrameMaker PostScript into a relocatable and hence usable form? >> >> Thanks for any guidance and/or pointers. >> >> Russel. > > >> ======================================================================= >> >> Dr Russel Winder >> >> Reader in Software Engineering >> Editor-in-Chief, Object Oriented Systems >> >> Information Systems Research Group >> Department of Computer Science Phone: +44 (0)171 380 7293 >> University College London Fax: +44 (0)171 387 1397 >> Gower Street EMail: R.Winder@cs.ucl.ac.uk >> London WC1E 6BT >> UK URL: http://www.cs.ucl.ac.uk/staff/R.Winder/ >> >> ======================================================================= > > You must make sure the applications export in EPS format. > > If you use raw PS, you are asking for trouble, because > PS drivers are allowed to use arbitrary PS code including the > forbidden-in-EPS operators that are troubling you. > > Most printer drivers have a check box this produce EPS output. > If you can't use EPS because the output is multi-page then you > are not using EPS, since that by definition is one page only. > > Berthold. > > > ------------------------------ > > From: Dominique de Waleffe > Date: Wed, 5 Jun 96 15:42:51 +0200 > Subject: Writing long formulas > > In Digital SRC report 119, Leslie Lamport presents a nice way to > format long mathematical formulas so that reading is a lot easier. > > Does anyone know a Latex package which implements those conventions? > > I'd be very interested. > > D. > - -- > Dominique de Waleffe > Mission Critical, Wijnegemhofstraat 199, B-3071 Erps-Kwerps (Belgium) > Phone: +32 2 759 95 60 Fax: +32 2 759 27 60 > email: ddw@acm.org, ddw@miscrit.be > PGP key fingerprint: F9 CC 23 74 44 62 7C F3 8C 12 DF 71 BB 60 54 98 > > > > ------------------------------ > > From: "Klaus M. Wendel" > Date: Tue, 11 Jun 1996 11:56:13 +0200 (MSZ) > Subject: Fonts in Latex2.09 > > Help! > > I am using LaTex Version 2.09 and i have to print a document in HELVETICA. > > Is there a (easy) way to do this? > > Thanks! > > - --------------------------------------------------- > Klaus M. Wendel always :-) > E-Mail: a9304890@unet.univie.ac.at > WWW: http://www.wu-wien.ac.at/usr/h93/h9304890/ > - --------------------------------------------------- > > > ------------------------------ > > From: David Simpson > Date: Mon, 17 Jun 1996 09:53:46 +0100 > Subject: [none] > > Does anybody know how to convert LaTeX(2e) files > to, dare I say, Word or Wordperfect documents? > Apologies to all tex fans for asking this, but > unfortunately I can't persuade all my PC-based colleagues > to switch to LateX, but I need to send them text > for reports. > > Thanks, Dave Simpson (david.simpson@dnmi.no) > > > ------------------------------ > > From: Rainer Schoepf > Date: Tue, 18 Jun 1996 11:14:13 +0200 (MET DST) > Subject: EMTeX no longer available from ftp.shsu.edu > > As ftp.shsu.edu is not properly managed, and the emTeX files there are > partially out of date, I removed emTeX at teh request of Eberhard > Mattes from ftp.shsu.edu. > > We are apologize for the inconvenience, but the CTAN team doesn't have > the resources to manage a site from across the atlantic. > > For the CTAN team > > - -- > Rainer Schvpf > Zentrum f|r Datenverarbeitung A point of view can be a dangerous > der Universitdt Mainz luxury when substituted for insight > Anselm-Franz-von-Bentzel-Weg 12 and understanding. > D-55099 Mainz > Germany Herbert Marshall McLuhan: > The Gutenberg Galaxy > > ------------------------------ > > End of TeXhax Digest V96 #8 > *************************** > > > About TeXhax... > > Please send contributions to: TeXhax@tex.ac.uk > > Subscription and unsubscription requests: > send a one line mail message to TeXhax-Request@tex.ac.uk > containing either subscribe texhax > or unsubscribe texhax > If you have problems with un/subscribing, please mail owner-texhax@nott.ac.uk > > To obtain the Frequently Asked Questions (FAQ) lists for TeX, send a > message with no subject to fileserv@shsu.edu, consisting of > SENDME FAQ > > For information on the TeX Users Group, please send a message to > TUG@TUG.org, or write TeX Users Group, 1850 Union Street, #1637 > San Francisco CA 94123 (phone: 1 415 982 8449, fax: 1 415 982 8559) > > Backnumbers of all the digests are stored in the Comprehensive TeX > Archive Network (CTAN) and can be retrieved on the Internet by > anonymous ftp. The hosts comprising CTAN include, among others, > ftp.dante.de (129.69.1.12) -- Germany > ftp.shsu.edu (192.92.115.10) -- USA > ftp.tex.ac.uk (128.232.1.87) -- UK > Please use your nearest server, to keep network load down. > The file /tex-archive/CTAN.sites on each of these hosts gives a > list of other sites which maintain full or partial mirrors of the CTAN. > Alternatively, finger ctan_us@ftp.shsu.edu for full details. > > TeXhax Digest back issues are filed below /tex-archive/digests/texhax/ > Keyword-In-Context indexes are filed in /tex-archive/digests/indexes/ > > A Hypermail version of TeXhax is also available on the World-Wide Web at URL > http://www.tex.ac.uk/tex-archive/digests/hyper/ > > \bye ------------------------------ End of TeXhax Digest V96 #9 *************************** About TeXhax... Please send contributions to: TeXhax@tex.ac.uk Subscription and unsubscription requests: send a one line mail message to TeXhax-Request@tex.ac.uk containing either subscribe texhax or unsubscribe texhax If you have problems with un/subscribing, please mail owner-texhax@nott.ac.uk To obtain the Frequently Asked Questions (FAQ) lists for TeX, send a message with no subject to fileserv@shsu.edu, consisting of SENDME FAQ For information on the TeX Users Group, please send a message to TUG@TUG.org, or write TeX Users Group, 1850 Union Street, #1637 San Francisco CA 94123 (phone: 1 415 982 8449, fax: 1 415 982 8559) Backnumbers of all the digests are stored in the Comprehensive TeX Archive Network (CTAN) and can be retrieved on the Internet by anonymous ftp. The hosts comprising CTAN include, among others, ftp.dante.de (129.69.1.12) -- Germany ftp.shsu.edu (192.92.115.10) -- USA ftp.tex.ac.uk (128.232.1.87) -- UK Please use your nearest server, to keep network load down. The file /tex-archive/CTAN.sites on each of these hosts gives a list of other sites which maintain full or partial mirrors of the CTAN. Alternatively, finger ctan_us@ftp.shsu.edu for full details. TeXhax Digest back issues are filed below /tex-archive/digests/texhax/ Keyword-In-Context indexes are filed in /tex-archive/digests/indexes/ A Hypermail version of TeXhax is also available on the World-Wide Web at URL http://www.tex.ac.uk/tex-archive/digests/hyper/ \bye From owner-texhax-digest@nottingham.ac.uk Thu Jul 11 18:28:57 1996 Flags: 000000000001 Received: from jess.ccc.nottingham.ac.uk (jess.ccc.nottingham.ac.uk [128.243.40.193]) by csc-sun.math.utah.edu (8.7.4/8.7.3) with ESMTP id SAA20834 for ; Thu, 11 Jul 1996 18:28:54 -0600 (MDT) Message-Id: <199607120028.SAA20834@csc-sun.math.utah.edu> Received: from nottingham.ac.uk by jess.ccc.nottingham.ac.uk id <12952-0@jess.ccc.nottingham.ac.uk>; Fri, 12 Jul 1996 01:14:05 +0100 From: Majordomo list server To: texhax-digest@nottingham.ac.uk Subject: TeXhax Digest V96 #10 Reply-To: TeXhax@tex.ac.uk Errors-To: owner-texhax-digest@nottingham.ac.uk Precedence: bulk Date: Fri, 12 Jul 1996 01:14:05 +0100 Sender: owner-texhax-digest@nottingham.ac.uk TeXhax Digest Friday, 12 July 1996 Volume 96 : Number 010 (incorporating UKTeX Digest) Today's Topics: Re: TeXhax Digest V96 #8 -- Helvetica Fonts in Latex2.09 wysiwyg Announcement NTG TeX & Graphics course Re: TeXhax postings to the TEX-L list New address METAFONT for Macintosh? TUGBoat 17(3) Chancellor Group (symbol = CHAG) ---------------------------------------------------------------------- From: "e-Floyd B. Hanson, UIC 312-413-2142" Date: Tue, 18 Jun 1996 12:09:45 CDT Subject: Re: TeXhax Digest V96 #8 -- Helvetica Fonts in Latex2.09 If you have psfonts, then a simple way to use helvetica is to use the style file helv.sty: - ---cut--- % latex style with helvetica text. \def\@mrm{phvr}% Helvetica - roman \def\@mit{phvro}% Helvetica -oblique \def\@msl{phvbo}% Helvetica - bold - oblique \def\@mbf{phvb}% Helvetica - bold \def\@mcsc{phvrc}% Helvetica - small caps \def\@mtt{pcrr}% courier \def\@mss{phvr}% helvetica \input psfonts.sty - ---cut--- > ------------------------------ > > From: "Klaus M. Wendel" > Date: Tue, 11 Jun 1996 11:56:13 +0200 (MSZ) > Subject: Fonts in Latex2.09 > > Help! > > I am using LaTex Version 2.09 and i have to print a document in HELVETICA. > > Is there a (easy) way to do this? > > Thanks! > > - --------------------------------------------------- ------------------------------ From: Hartmut Peters Date: Tue, 18 Jun 1996 18:34:47 -0400 (EDT) Subject: wysiwyg Dear fellow TeXers, I am having a paper in the mill at Academic Press in the UK, and they only do wysiwyg programs like word, yuk! My manuscript is in LaTeX both on Unix and on a Mac under Textures. Does anybody know of a way to export the text to wysiwyg programs? It wouldn't be a problem if the formatting information got lost as long as the text came across. (Excalibur has a parser that throws out the LaTeX/TeX commands...) The opposite way apparently works, but I haven't seen TeX --> wysiwyg, yet. Well, if not, the Academic Press folks may have to digest the LaTeX files or retype it (in which case I get to find all the typos...) I sure wish they could handle something more professional than "Word"... Best regards from Lawn Guyland in the strange State of New York --> :::::::::::::::::::::::::::::::::::::::::::::::::::::::::::::::::::::::::::::::: HARTMUT PETERS | E-MAIL: hartmut.peters@sunysb.edu (Assistant Professor) | OR: peters@sunhp.msrc.sunysb.edu | BITNET: hpeters@sbccmail.bitnet Marine Sciences Research Center | All-In-One/Stony Brook: hpeters State University of New York at ... | Stony Brook, NY 11794-5000 | TELEPHONE: +516 632-8682 U.S.A. | TELEFAX: +516 632-8820 :::::::::::::::::::::::::::::::::::::::::::::::::::::::::::::::::::::::::::::::: ------------------------------ From: cgl@rc.service.rug.nl (Kees van der Laan) Date: Mon, 24 Jun 1996 20:34:55 +0200 Subject: Announcement NTG TeX & Graphics course ---------------------------------------------------------- |Announcement NTG's one-day low-budget no-nonsense course| | | | (La)TeX and Graphics | ---------------------------------------------------------- What? MetaPost, PostScript, mftoeps... (no font design) Language? English Teacher? Boguslaw Jackowksi Where? University of Utrecht When? The day before *or* after NTG's fall meeting of 24 Oct. (To be announced late August) Costs? Fl 50 members of NTG or other LUGs; non-members Fl 150 Subsription? Treasurer NTG, Giro 1306238, Eindhoven (Do mention TeX and Graphics course) Information? Kees van der Laan, cgl@rc.service.rug.nl Literature? Jackowksi's METAFONT booklet will be in English available. Have a look at Hobby's CSTR 162, A user manual for MetaPost, from netlib@research.att.com with message send 162 from research/cstr or copy it from NTG's 4AllTeX CD-ROM When the number of subscriptions is insufficient of 7 September---ultimate date for registration--- the course will be cancelled (and money refunded). - ---Kees--- Sent to: TeXhax@tex.ac.uk (earlier Dutch announcement to tex-nl@nic.surfnet.nl) cc: TeX-D-L@vm.gmd.de, gut@ens.fr, uktex@tex.ac.uk, metafont@ens.fr, TUG@TUG.org, info-tex@shsu.edu ------------------------------ From: Melvin Fitting Date: Fri, 05 Jul 96 09:53:55 EDT Subject: Re: TeXhax postings to the TEX-L list - --------------------------------- | New e-mail address | | fitting@alpha.lehman.cuny.edu | | Please use from now on | - --------------------------------- ------------------------------ From: Melvin Fitting Date: Fri, 05 Jul 96 09:54:34 EDT Subject: New address - --------------------------------- | New e-mail address | | fitting@alpha.lehman.cuny.edu | | Please use from now on | - --------------------------------- ------------------------------ From: Tom Bryan Date: Fri, 5 Jul 96 10:29:29 PDT Subject: METAFONT for Macintosh? Has anyone had success in running METAFONT on a Macintosh? I have a Macintosh PowerBook 5300c, system 7.5. I am using the public domain METAFONT version 0.66 from the Bluesky ftp site, and it is not fully operational. Is there a better METAFONT for Macintosh available? Once the MF file is written, how does one install a font on the Macintosh so that Bluesky's Textures can display the font on the screen and print? Has anyone had success getting "penlabels" to work? Does anyone know how to produce output with grid lines and penlabels like those in Donald Knuth's "Computer Modern Typefaces, Computers & Typesetting volume E"? For some reason, I have not received a TeXhax digest in a while even though I am still on the list, so please reply directly to me in addition to posting to the list. Sincerely, Tom Bryan E-Systems, Goleta Division One South Los Carneros, Goleta, CA 93117-3197, USA Email: tbryan@esd.ray.com Tel: (805) 967-5511 ext. 2974 FAX: (805) 964-0470 ------------------------------ From: Mimi Burbank Date: Fri, 5 Jul 1996 16:14:04 -0500 (EDT) Subject: TUGBoat 17(3) Dear colleagues, Below are the contents of the Proceedings Issue of TUGboat for 1996. TUGboat Volume 17, Number 2 / June 1996 ================================ Opening Address Michel Goossens Opening Words by the President 91 %%%%%%%%%%% Fonts Karel Piska Cyrillic alphabets 92 J\"org Knappen The dc fonts 1.3: Move towards stability and completeness 99 Fukui Rei TIPA: A system for processing phonetic symbols in LaTeX A.S. Berdnikov Computer Modern Typefaces as Multiple Master Fonts 115 A.S. Berdnikov VFComb 1.3 --- the program which simplifies virtual font management 120 %%%%%%%%%%% Encoding and Multilingual Support Yannis Haralambous \Omega Times and \Omega Helvetica fonts under development: Step ~One 126 Richard J.~Kinch Extending TeX for Unicode 147 L.N. Znamenskaya and S.V. Znamenskii Russian encoding plurality problem and a new Cyrillic font set 161 Peter~A.~Ovchenkov Cyrillic TeX files: interplatform portability 166 Michael M. Vinogradov A user-friendly multi-function TeX interface based on Multi-Edit 172 Olga~G. Lapko Full Cyrillic: How many languages? 174 %%%%%%%%%%% TeX Systems John Plaice and Yannis Haralambous The latest developments in \Omega 181 Dag Langmyhr StarTeX --- a TeX for beginners 184 Gabriel Valiente Feruglio Do journals honor LaTeX submissions? 191 Sergei V. Znamenskii and Denis E. Leinartas A new approach to the TeX-related programs: A user-friendly interface 200 Ivan G. Vsesvetsky The strait gate to TeX 204 Laurent Siebenmann DVI-based electronic publication 206 Kees~van~der~Laan BLUe's format --- the off-off alternative 215 %%%%%%%%%%% Graphics Kees~van~der~Laan Turtle graphics and TeX -- a child can do it 222 A.S. Berdnikov, O.A. Grineva and S.B. Turtia Some useful macros which extend the LaTeX picture environment 229 %%%%%%%%%%% News & Announcements Mimi Burbank Production notes 90 Calendar 233 %%%%%%%%%%% TUG Business< Institutional members 234 %%%%%%%%%%% Advertisements TeX consulting and production services 235 ====== end of file ------------------------------ From: chag@moneyworld.com Date: Fri, 12 Jul 1996 01:06:33 +0100 Subject: Chancellor Group (symbol = CHAG) http://chancellor.stockpick.com Chancellor Group, Inc. (symbol CHAG) just reported big quarterly earnings. SGA Goldstar issued a "buy" recommendation. I understand other investment advisors are looking to recommend CHAG. The company has a strong book value. The short sellers need to cover. This looks like a good situation to me. What do you think? They are located at: http://chancellor.stockpick.com Bob Williams, 206-269-0846 To terminate from my Investment Opportunities, Reply to chag@moneyworld.com with "remove" in the subject field. ------------------------------ End of TeXhax Digest V96 #10 **************************** About TeXhax... Please send contributions to: TeXhax@tex.ac.uk Subscription and unsubscription requests: send a one line mail message to TeXhax-Request@tex.ac.uk containing either subscribe texhax or unsubscribe texhax If you have problems with un/subscribing, please mail owner-texhax@nott.ac.uk To obtain the Frequently Asked Questions (FAQ) lists for TeX, send a message with no subject to fileserv@shsu.edu, consisting of SENDME FAQ For information on the TeX Users Group, please send a message to TUG@TUG.org, or write TeX Users Group, 1850 Union Street, #1637 San Francisco CA 94123 (phone: 1 415 982 8449, fax: 1 415 982 8559) Backnumbers of all the digests are stored in the Comprehensive TeX Archive Network (CTAN) and can be retrieved on the Internet by anonymous ftp. The hosts comprising CTAN include, among others, ftp.dante.de (129.69.1.12) -- Germany ftp.shsu.edu (192.92.115.10) -- USA ftp.tex.ac.uk (128.232.1.87) -- UK Please use your nearest server, to keep network load down. The file /tex-archive/CTAN.sites on each of these hosts gives a list of other sites which maintain full or partial mirrors of the CTAN. Alternatively, finger ctan_us@ftp.shsu.edu for full details. TeXhax Digest back issues are filed below /tex-archive/digests/texhax/ Keyword-In-Context indexes are filed in /tex-archive/digests/indexes/ A Hypermail version of TeXhax is also available on the World-Wide Web at URL http://www.tex.ac.uk/tex-archive/digests/hyper/ \bye From owner-texhax-digest@nottingham.ac.uk Wed Jul 17 17:04:11 1996 Flags: 000000000001 Received: from jess.ccc.nottingham.ac.uk (jess.ccc.nottingham.ac.uk [128.243.40.193]) by csc-sun.math.utah.edu (8.7.4/8.7.3) with ESMTP id RAA10524 for ; Wed, 17 Jul 1996 17:03:54 -0600 (MDT) Message-Id: <199607172303.RAA10524@csc-sun.math.utah.edu> Received: from nottingham.ac.uk by jess.ccc.nottingham.ac.uk id <07474-0@jess.ccc.nottingham.ac.uk>; Wed, 17 Jul 1996 23:44:30 +0100 From: Majordomo list server To: texhax-digest@nottingham.ac.uk Subject: TeXhax Digest V96 #11 Reply-To: TeXhax@tex.ac.uk Errors-To: owner-texhax-digest@nottingham.ac.uk Precedence: bulk Date: Wed, 17 Jul 1996 23:44:30 +0100 Sender: owner-texhax-digest@nottingham.ac.uk TeXhax Digest Wednesday, 17 July 1996 Volume 96 : Number 011 (incorporating UKTeX Digest) Today's Topics: Re: METAFONT for Macintosh? Re: TeXhax Digest V96 #10 Re: wysiwyg (TeXhax Digest V96 #10) Re: TeXhax Digest V96 #10 Metafont on Mac Re: Metafont on Mac ---------------------------------------------------------------------- From: phadke@oceaneng.eng.hawaii.edu (Amal Phadke) Date: Thu, 11 Jul 1996 16:23:54 -1000 Subject: Re: METAFONT for Macintosh? > From: Tom Bryan > Date: Fri, 5 Jul 96 10:29:29 PDT > Subject: METAFONT for Macintosh? > > Has anyone had success in running METAFONT on a Macintosh? > > I have a Macintosh PowerBook 5300c, system 7.5. > I am using the public domain METAFONT version 0.66 from the > Bluesky ftp site, and it is not fully operational. > Is there a better METAFONT for Macintosh available? > > Sincerely, > Tom Bryan > > E-Systems, Goleta Division > One South Los Carneros, Goleta, CA 93117-3197, USA > Email: tbryan@esd.ray.com Tel: (805) 967-5511 ext. 2974 > FAX: (805) 964-0470 > Tom: OzTeX 2.0.1, Andrew Trevorrow's Macintosh shareware version of TeX now has its own METAFONT program OzMF. It has worked flawlessly for me so far. OzTeX is available at all CTAN sites under "/tex-archive/systems/mac/oztex" directory. Cheers - --- Amal Phadke Department of Ocean Engineering, School of Ocean and Earth Sciences & Technology University of Hawaii at Manoa 2540 Dole street , Holmes Hall 408B Honolulu, HI 96822, USA Tel: (808) 956-8198 Fax: (808) 956-3498 e-mail: phadke@oceaneng.eng.hawaii.edu WWW Page : http://oceaneng.eng.hawaii.edu/~phadke ------------------------------ From: Piet van Oostrum Date: Fri, 12 Jul 1996 11:56:52 +0200 Subject: Re: TeXhax Digest V96 #10 The following message is a courtesy copy of an article that has been posted as well. >>>>> Hartmut Peters (HP) writes: HP> Dear fellow TeXers, HP> I am having a paper in the mill at Academic Press in the UK, and they only HP> do wysiwyg programs like word, yuk! My manuscript is in LaTeX both on Unix HP> and on a Mac under Textures. Does anybody know of a way to export the text HP> to wysiwyg programs? It wouldn't be a problem if the formatting HP> information got lost as long as the text came across. (Excalibur has a HP> parser that throws out the LaTeX/TeX commands...) HP> The opposite way apparently works, but I haven't seen TeX --> wysiwyg, HP> yet. Well, if not, the Academic Press folks may have to digest the LaTeX HP> files or retype it (in which case I get to find all the typos...) I sure HP> wish they could handle something more professional than "Word"... Try tex2rtf (not latex2rtf), from the support directory on CTAN. It produces RTF which can be read both by Wordpervert and Microsloth Word, and probably plenty of others, too. If that doesn't work you could try detex, but it may throw away too much. Or latex it, and push it through dvi2tty. - -- Piet van Oostrum URL: http://www.cs.ruu.nl/~piet [PGP] ------------------------------ From: piet@cs.ruu.nl (Piet van Oostrum) Date: Fri, 12 Jul 1996 12:09:21 +0200 Subject: Re: wysiwyg (TeXhax Digest V96 #10) The following message is a courtesy copy of an article that has been posted as well. >>>>> Hartmut Peters (HP) writes: HP> Dear fellow TeXers, HP> I am having a paper in the mill at Academic Press in the UK, and they only HP> do wysiwyg programs like word, yuk! My manuscript is in LaTeX both on Unix HP> and on a Mac under Textures. Does anybody know of a way to export the text HP> to wysiwyg programs? It wouldn't be a problem if the formatting HP> information got lost as long as the text came across. (Excalibur has a HP> parser that throws out the LaTeX/TeX commands...) HP> The opposite way apparently works, but I haven't seen TeX --> wysiwyg, HP> yet. Well, if not, the Academic Press folks may have to digest the LaTeX HP> files or retype it (in which case I get to find all the typos...) I sure HP> wish they could handle something more professional than "Word"... Try tex2rtf (not latex2rtf), from the support directory on CTAN. It produces RTF which can be read both by Wordpervert and Microsloth Word, and probably plenty of others, too. If that doesn't work you could try detex, but it may throw away too much. Or latex it, and push it through dvi2tty. - -- Piet van Oostrum URL: http://www.cs.ruu.nl/~piet [PGP] ------------------------------ From: John Burt Date: Fri, 12 Jul 1996 11:18:02 -0500 (EST) Subject: Re: TeXhax Digest V96 #10 Yes, I have had success running Metafont on a Mac. The one I use is the Metafont that is a part of the CMacTeX package. It makes the standard .tfm and .gf files, and you run gftopk on the .gf file to generate a .pk file. Running Metafont is not trivial, but this Metafont behaves exactly like the metafonts in the unix and vms worlds. It is shareware, but you get the whole CMacTeX package, which is great, for your money. You can download the most recent version from the author's home page. I don't have his URL handy, but I think it's http://math.tamu.edu/~tkiffe/cmactex.html The author's name is Tom Kiffe. One note: your TeX must use standard .pk files (CMacTeX and OzTeX do---in fact the latter uses Kiffe's Metafont as well). Some systems---I think Te textures among them---don't use standard .pk files. John Burt burt@binah.cc.brandeis.edu ------------------------------ From: campbell@beloit.edu (Paul J. Campbell) Date: Fri, 12 Jul 1996 13:19:18 -0600 Subject: Metafont on Mac I too have been experimenting with Metafont 0.66 on the Mac (acutally, when it runs, it says it is 0.65). I'm glad you asked your question, so I can describe for my own benefit---before I forget!---what works. There is a file metafont-for-beginners on CTAN (I think it is in /doc), but I have not seen anything specific to the Mac. Last week I had my first success with Metafont. I was able to take a Metafont description of an .mf file and turn it into a font suitcase that works with any application. The key problem is that .mf files cannot be processed without some additions. I used the program shell mode=laserwriter; % Metafont checks mfinputs:local.mf for all mode_def's; % put here the option that fits your printer mag=magstep0; % could be magstep1, or 0.5,2,3,4,5 as well %screenstrokes; % could be screenchars, for per character viewing, but works faster this way input hcyr % this is the .mf file to process; add more input lines as needed bye To the original .mf file I added the following introductory lines: font_size 10pt#; % the "design size" of this font; can be whatever you like ht#:=10pt#; % height of characters xgap#:=0.6pt#; % horizontal adjustment (I just guessed at this) If you don't put in lines like those, everything appears to work, Metafont says that it writes the fonts to files, and indeed it creates files---empty ones! You process the program shell file with Metafont with the settings Make Suitcase and (for me, by default) Plain, by selecting Metafont on the Metafont menu. Metafont adds the font to the MFfonts suitcase and creates an MFmetrics suitcase (you can see the latter and you can double-click the former to examine the contents---that's how I found out I wasn't getting anything). You need the latter for use with TeX but the former will work with any program (Word, etc.) if you put it into the System folder. The application program (e.g., Word) will scale the font (with jaggies) to any size; if you want nice looking characters, process the .mf file to make fonts at all the sizes you will use. I was just making a font of ancient Croatian script for a couple of students here, but part of my reason for exploring all this is an old desire (and a new need) to be able to take fonts for IBM (.pfb, .pfa, .pfm files) and make them into Mac bitmapped (or preferably PS) fonts. After exploring lots of software on CTAN last week, I am convinced that this can be done but (as far as I know) only by making a detour to using some Unix programs along the way. A promising new port of TeX, LaTeX, Metafont (0.66), BibTex, and Metapost(!) for the Mac was promulgated this April. Called cmactex25, it is available at CTAN (it's shareware, $35). CMacTeX handles afm, pfa, pfb, and tfm files directly and there is no need to convert them into a Mac-specific format. I have explored only its font utilities so far. Paul J. Campbell Mathematics and Computer Science Beloit College 700 College St. Beloit, WI 53511 (608) 363-2007 (ofc) 362-2805 (res) 363-2718 (fax) campbell@beloit.edu ------------------------------ From: Tom Bryan Date: Wed, 17 Jul 96 15:41:45 PDT Subject: Re: Metafont on Mac Dear Paul, Thank you for your advice on using Metafont 0.66 on the Mac. I tried your advice and got the Blue Sky version to work for the first time. However, I was still dissatisfied with the deficiencies of the Blue Sky Metafont. I received two other replies (enclosed) that led me to download and try OzTeX, DirectTeX, and CMacTeX. I only have a few days of experience with these other packages, but thus far I like the OzTeX the best and it replicates the examples in Knuth's MetaFont book without any problems (something I couldn't do in the Blue Sky version). Sincerely, Tom - -- Dr Thomas A Bryan E-Systems, Goleta Division One South Los Carneros, Goleta, California, USA 93117-3197, USA Email: tbryan@esd.ray.com Tel: (805) 967-5511 ext. 2974 FAX: (805) 964-0470 Home: 275 Savona Avenue, Goleta, California, 93117, USA (805) 968-1264 Return-Path: <@ESDSVR.esd.ray.com:Alun.J.Carr@ucd.ie> Date: Fri, 12 Jul 1996 18:22:47 +0100 From: Alun.J.Carr@ucd.ie (Dr Alun J. Carr) Subject: Re: METAFONT for Macintosh? X-Sender: ajcarr@pop3.ucd.ie To: tbryan@esd.ray.com Mime-Version: 1.0 Content-Type: text/plain; charset="us-ascii" Content-Transfer-Encoding: 7BIT Tom, >Has anyone had success in running METAFONT on a Macintosh? Well, I haven't even tried the Bluesky one, which is obsolete, to say the least. All the `mainstream' TeX implementations for the Mac (OzTeX, DirectTeX Pro, and CMacTeX) come with their own METAFONT implementations, which work extremely well, with the DVI previewers/printer drivers automatically calling METAFONT to generate missing fonts. Textures seems to be primarily Postscript font based, rather than bitmap-based, so METAFONT is a bit of an anomaly in a Textures system. I know only one Textures user here at UCD, and even he abandoned it a while back, and started using DirectTeX Pro (which works like a dream in conjunction with the Alpha text editor and the Excalibur spelling-checker). You can find the latest releases of OzTeX, CMacTeX and DirectTeX Pro, with their METAFONTs at the UK or German CTAN sites (the US CTAN is pretty-much defunct): Hope this helps. Alun - -- Dr Alun J. CARR Phone: +353-1-7061989 Mechanical Engineering Dept. +353-1-2693244 x1989 University College Dublin Fax: +353-1-2830534 Belfield E-mail: Dublin 4 WWW: Ireland Return-Path: <@ESDSVR.esd.ray.com:phadke@oceaneng.eng.hawaii.edu> Mime-Version: 1.0 Content-Type: text/plain; charset="us-ascii" Date: Thu, 11 Jul 1996 16:23:54 -1000 To: TeXhax@tex.ac.uk From: phadke@oceaneng.eng.hawaii.edu (Amal Phadke) Subject: Re: METAFONT for Macintosh? Cc: tbryan@esd.ray.com > From: Tom Bryan > Date: Fri, 5 Jul 96 10:29:29 PDT > Subject: METAFONT for Macintosh? > > Has anyone had success in running METAFONT on a Macintosh? > > I have a Macintosh PowerBook 5300c, system 7.5. > I am using the public domain METAFONT version 0.66 from the > Bluesky ftp site, and it is not fully operational. > Is there a better METAFONT for Macintosh available? > > Sincerely, > Tom Bryan > > E-Systems, Goleta Division > One South Los Carneros, Goleta, CA 93117-3197, USA > Email: tbryan@esd.ray.com Tel: (805) 967-5511 ext. 2974 > FAX: (805) 964-0470 > Tom: OzTeX 2.0.1, Andrew Trevorrow's Macintosh shareware version of TeX now has its own METAFONT program OzMF. It has worked flawlessly for me so far. OzTeX is available at all CTAN sites under "/tex-archive/systems/mac/oztex" directory. Cheers - --- Amal Phadke Department of Ocean Engineering, School of Ocean and Earth Sciences & Technology University of Hawaii at Manoa 2540 Dole street , Holmes Hall 408B Honolulu, HI 96822, USA Tel: (808) 956-8198 Fax: (808) 956-3498 e-mail: phadke@oceaneng.eng.hawaii.edu WWW Page : http://oceaneng.eng.hawaii.edu/~phadke ------------------------------ End of TeXhax Digest V96 #11 **************************** About TeXhax... Please send contributions to: TeXhax@tex.ac.uk Subscription and unsubscription requests: send a one line mail message to TeXhax-Request@tex.ac.uk containing either subscribe texhax or unsubscribe texhax If you have problems with un/subscribing, please mail owner-texhax@nott.ac.uk To obtain the Frequently Asked Questions (FAQ) lists for TeX, send a message with no subject to fileserv@shsu.edu, consisting of SENDME FAQ For information on the TeX Users Group, please send a message to TUG@TUG.org, or write TeX Users Group, 1850 Union Street, #1637 San Francisco CA 94123 (phone: 1 415 982 8449, fax: 1 415 982 8559) Backnumbers of all the digests are stored in the Comprehensive TeX Archive Network (CTAN) and can be retrieved on the Internet by anonymous ftp. The hosts comprising CTAN include, among others, ftp.dante.de (129.69.1.12) -- Germany ftp.shsu.edu (192.92.115.10) -- USA ftp.tex.ac.uk (128.232.1.87) -- UK Please use your nearest server, to keep network load down. The file /tex-archive/CTAN.sites on each of these hosts gives a list of other sites which maintain full or partial mirrors of the CTAN. Alternatively, finger ctan_us@ftp.shsu.edu for full details. TeXhax Digest back issues are filed below /tex-archive/digests/texhax/ Keyword-In-Context indexes are filed in /tex-archive/digests/indexes/ A Hypermail version of TeXhax is also available on the World-Wide Web at URL http://www.tex.ac.uk/tex-archive/digests/hyper/ \bye From texhax-digest-outgoing-request@nottingham.ac.uk Thu Aug 22 03:12:40 1996 Flags: 000000000001 Received: from jess.ccc.nottingham.ac.uk (jess.ccc.nottingham.ac.uk [128.243.40.193]) by csc-sun.math.utah.edu (8.7.4/8.7.3) with ESMTP id DAA20982 for ; Thu, 22 Aug 1996 03:12:37 -0600 (MDT) Message-Id: <199608220912.DAA20982@csc-sun.math.utah.edu> Received: from nottingham.ac.uk by jess.ccc.nottingham.ac.uk id <14654-0@jess.ccc.nottingham.ac.uk>; Thu, 22 Aug 1996 09:44:45 +0100 From: Majordomo list server To: texhax-digest@nottingham.ac.uk Subject: TeXhax Digest V96 #12 Reply-To: TeXhax@tex.ac.uk Errors-To: owner-texhax-digest@nottingham.ac.uk Precedence: bulk Date: Thu, 22 Aug 1996 09:44:45 +0100 Sender: owner-texhax-digest@nottingham.ac.uk TeXhax Digest Thursday, 22 August 1996 Volume 96 : Number 012 (incorporating UKTeX Digest) Today's Topics: Re: METAFONT for Macintosh? Migration of CTAN-related mailing lists Matrices T.V. Raman in the news ANNOUCEMENT: Sauter-2.4 single quotes using the times postscript fonts??? How do you get equal variable-sized delimiters? Coversion of DVI file to ASCII File DEK in the news ---------------------------------------------------------------------- From: Tom Bryan Date: Wed, 17 Jul 96 15:50:23 PDT Subject: Re: METAFONT for Macintosh? Amal: Thank you for your advice on running METAFONT on the Macintosh. I downloaded OzTeX and it is working very well for me. I had been beating myself over the head with the Blue Sky version for weeks trying to get it to work properly. You saved me a great deal of time. Cheers, Tom - -- Dr Thomas A Bryan E-Systems, Goleta Division One South Los Carneros, Goleta, California, USA 93117-3197, USA Email: tbryan@esd.ray.com Tel: (805) 967-5511 ext. 2974 FAX: (805) 964-0470 Home: 275 Savona Avenue, Goleta, California, 93117, USA (805) 968-1264 ------------------------------ From: Rainer Schoepf Date: Thu, 18 Jul 1996 19:01:13 +0200 (MET DST) Subject: Migration of CTAN-related mailing lists The CTAN-related mailing lists CTAN@shsu.edu CTAN-ANN@shsu.edu have now been migrated to a new mailing list server, located at Heidelberg, Germany. These lists can now be reached as CTAN@urz.Uni-Heidelberg.de CTAN-ANN@urz.Uni-Heidelberg.de Requests for subscription/deletion must be sent to LISTSERV@urz.Uni-Heidelberg.de For the CTAN team - -- Rainer Schvpf Zentrum f|r Datenverarbeitung A point of view can be a dangerous der Universitdt Mainz luxury when substituted for insight Anselm-Franz-von-Bentzel-Weg 12 and understanding. D-55099 Mainz Germany Herbert Marshall McLuhan: The Gutenberg Galaxy ------------------------------ From: Barry Tesman Date: Fri, 26 Jul 1996 09:37:43 -0400 (EDT) Subject: Matrices I am trying to produce the following "labelled" matrix with latex: B1 B2 B3 / \ a1 | 8 10 4 | A = a2 | 11 5 18 | a3 | 27 6 18 | \ / I tried to use array/tabbing/tabular but am having trouble placing the left and right large parentheses around the matrix but not the labels. Any ideas would be most helpful. Thanks, Barry Tesman < tesman@dickinson.edu > Dickinson College Carlisle, PA, USA ------------------------------ From: "Nelson H. F. Beebe" Date: Mon, 19 Aug 1996 08:53:20 -0600 (MDT) Subject: T.V. Raman in the news You might care to look at this just-appeared profile of TUGboat author and TUG conference participant T. V. Raman: @String{j-SCI-AMER = "Scientific American"} @Article{Gibbs:1996:PVR, author = "W. Wayt Gibbs", title = "Profile: T. V. Raman: Envisioning Speech", journal = j-SCI-AMER, volume = "275", number = "3", pages = "52, 54", month = nov, year = "1996", acknowledgement = ack-nhfb, bibdate = "Mon Aug 19 08:48:09 1996", } Curiously, this article shows a single equation, which seems to violate the Scientific American practice of eschewing equations. ======================================================================== Nelson H. F. Beebe Tel: +1 801 581 5254 Center for Scientific Computing FAX: +1 801 581 4148 Department of Mathematics, 105 JWB Internet: beebe@math.utah.edu University of Utah URL: http://www.math.utah.edu/~beebe Salt Lake City, UT 84112, USA ======================================================================== ------------------------------ From: KNAPPEN@VKPMZD.kph.Uni-Mainz.DE Date: Sun, 21 Jul 1996 16:31:56 +0100 Subject: ANNOUCEMENT: Sauter-2.4 I have uploaded a new Sauter release to the CTAN archives in directory tex-archive/fonts/cm/sauter24/ and it will propagate to the mirrors soon. Attached is the release file. J"org Knappen. %*%*%*%*%*%*%*%*%*%*%*%*%*%*%*%*%*%*%*%*%*%*%*%*%*%*%*%*%*%*%*%*%*%*%*%*%*%*%*%* The release 2.4 of the Sauter tools to generate true sized fonts contains the following news: * Support for LaTeX2e 1996/06/01 * Fixes to cmcsc fonts * Fix to the capital S in cmmi fonts * Sauterised versions of AMS symbol fonts, bbm fonts and rsfs fonts contributed by Hubert Holin If you already have version 2.3 of the Sauter fonts, please regenerate the cmcsc fonts for sizes => 10pt, there was a bug affecting the depth of the letter `q'. Please recalculate the cmmi fonts for sizes >12pt, the capital S will look much better afterwards. Send any questions, comments and suggestions to J"org Knappen knappen@vkpmzd.kph.uni-mainz.de ------------------------------ From: coleman@natasha.phil.uregina.ca (Robert Coleman) Date: Mon, 22 Jul 1996 16:09:17 -0600 (CST) Subject: single quotes using the times postscript fonts??? I have installed the times.sty package that uses the psfonts courier, helvetica and times in LaTeX2e. In the times font ptmr8r, the usual positions for the left single quote "60 and for the right single quote "27 contain instead of the symbols found in the font cmr a symbol that looks like a grave accent and a symbol that looks like a vertical prime, respectively. How is one to get matched single quotes? There is an older font rptmr that has the usual single quotes in those positions. Is there some way to use the font ptmr8r and still get the usual single quotes, for example, by means of some trick using virtual fonts and rptmr? Thanks, Robert - -- Robert Alan Coleman Department of Physics University of Regina Regina, Saskatchewan Canada S4S 0A2 Tel: (306) 585-4260 Fax: (306) 585-4894 email: coleman@cas.uregina.ca ********end message*********************** ------------------------------ From: holliegp@eee.bham.ac.uk (Garry Hollier) Date: Wed, 21 Aug 1996 12:21:42 +0100 Subject: How do you get equal variable-sized delimiters? In a LaTeX (I'm using LaTeX2e <1994/06/01> patch level 2) expression like \begin{eqnarray} \left %some stuff \right.\\ \left. %some more stuff \right \end{eqnarray} how do you ensure that the resulting 's are the same size? ------------------------------ From: "ABDOL S. SOOFI, ECONOMICS, UW-PLATTEVILLE; HTTP://VMS.WWW.UWPLATT.EDU/~SOOFI" Date: Wed, 21 Aug 1996 11:00:59 -0500 (CDT) Subject: Coversion of DVI file to ASCII File Greetings to all, Is there a way to convert a filename.dvi to its LaTeX source file (ASCII file)? I am in a peculiar situation. I have the dvi version of a revised file, but its revised LaTeX source file is missing? Also, what is the meaning of ``dvi file is corrupt''? What are the possible sources of this problem? Thanks in advance for any insights. - --Abdol ------------------------------ From: "Nelson H. F. Beebe" Date: Fri, 16 Aug 1996 09:54:23 -0600 (MDT) Subject: DEK in the news The September 1996 issue of Dr. Dobb's Journal has an editorial on p. 6 which begins: It was a pleasant surprise to open the mail and read that Donald Knuth, the author of the three-volume {\em The Art of Computer Programming}, is the recipient of the Inamori Foundation's 1996 Kyoto Prize in the category of Advanced Technology. Considering that a check for $460,000 accompanies his wall plaque, Dr. Knuth is probably more pleased than I. According to Kazuo Inamori, president of the Inamori Foundation and chairman of Kyocera, Knuth was given Japan's highest private award for his contributions to the betterment of humankind. In addition to {\em The Art of Computer Programming}, Knuth (who was featured in an April 1996 DDJ interview) is the creator of the TeX document-preparation system, Metafont font-design system, and LR parser and attribute grammar. TeX has been described as the most important achievement in publishing since Gutenberg's moveable type. Previous Kyoto Advanced Technology laureates include John McCarthy of artificial-intelligence fame, and George Gray, developer of the liquid-crystal display. Please join me in hearty congratulations to Dr. Knuth. ... Jonathan Erickson Editor-in-chief ======================================================================== Nelson H. F. Beebe Tel: +1 801 581 5254 Center for Scientific Computing FAX: +1 801 581 4148 Department of Mathematics, 105 JWB Internet: beebe@math.utah.edu University of Utah URL: http://www.math.utah.edu/~beebe Salt Lake City, UT 84112, USA ======================================================================== ------------------------------ End of TeXhax Digest V96 #12 **************************** About TeXhax... Please send contributions to: TeXhax@tex.ac.uk Subscription and unsubscription requests: send a one line mail message to TeXhax-Request@tex.ac.uk containing either subscribe texhax or unsubscribe texhax If you have problems with un/subscribing, please mail owner-texhax@nott.ac.uk To obtain the Frequently Asked Questions (FAQ) lists for TeX, send a message with no subject to fileserv@shsu.edu, consisting of SENDME FAQ For information on the TeX Users Group, please send a message to TUG@TUG.org, or write TeX Users Group, 1850 Union Street, #1637 San Francisco CA 94123 (phone: 1 415 982 8449, fax: 1 415 982 8559) Backnumbers of all the digests are stored in the Comprehensive TeX Archive Network (CTAN) and can be retrieved on the Internet by anonymous ftp. The hosts comprising CTAN include, among others, ftp.dante.de (129.69.1.12) -- Germany ftp.shsu.edu (192.92.115.10) -- USA ftp.tex.ac.uk (128.232.1.87) -- UK Please use your nearest server, to keep network load down. The file /tex-archive/CTAN.sites on each of these hosts gives a list of other sites which maintain full or partial mirrors of the CTAN. Alternatively, finger ctan_us@ftp.shsu.edu for full details. TeXhax Digest back issues are filed below /tex-archive/digests/texhax/ Keyword-In-Context indexes are filed in /tex-archive/digests/indexes/ A Hypermail version of TeXhax is also available on the World-Wide Web at URL http://www.tex.ac.uk/tex-archive/digests/hyper/ \bye From owner-texhax-digest@nottingham.ac.uk Tue Oct 15 05:41:00 1996 Flags: 000000000001 Received: from jess.ccc.nottingham.ac.uk (jess.ccc.nottingham.ac.uk [128.243.40.193]) by csc-sun.math.utah.edu (8.7.6/8.7.3) with ESMTP id FAA27278 for ; Tue, 15 Oct 1996 05:40:52 -0600 (MDT) Message-Id: <199610151140.FAA27278@csc-sun.math.utah.edu> Received: from nottingham.ac.uk by jess.ccc.nottingham.ac.uk id <14694-0@jess.ccc.nottingham.ac.uk>; Tue, 15 Oct 1996 11:42:19 +0100 From: Majordomo list server To: texhax-digest@nottingham.ac.uk Subject: TeXhax Digest V96 #13 Reply-To: TeXhax@tex.ac.uk Errors-To: owner-texhax-digest@nottingham.ac.uk Precedence: bulk Date: Tue, 15 Oct 1996 11:42:19 +0100 Sender: owner-texhax-digest@nottingham.ac.uk TeXhax Digest Tuesday, 15 October 1996 Volume 96 : Number 013 (incorporating UKTeX Digest) Today's Topics: Answers Re:TeXhax Digest V96 #12 Re: TeXhax Digest V96 #12 (Fwd) Repeated announcement T&G course NTG Memory question ispell problems Re: word to latex converter Announce: xtem X11-TeX-Menu 5.23 ---------------------------------------------------------------------- From: Jose Manuel Souto Menendez Date: Fri, 4 Oct 1996 19:14:51 +0100 (WET DST) Subject: Answers I am writing a book with Latex and I would like to know how it is posible to write the answers of some of the exercises in such a way that that they appear at the end of the book. Are there any macros to solve this problem? (I am using AMSLaTeX, and with LaTeX2e). Thank you very much, Jose M. Souto ------------------------------ From: kletzing@totcon.com (Dennis Kletzing) Date: Fri, 23 Aug 1996 12:47:39 -0400 Subject: Re:TeXhax Digest V96 #12 Barry: I just noticed your post to TeXhax. Did you ever get a response? The \bordermatrix macro in the Texbook will do just what you want. If you don't have it I'll be glad to send it along. >In article <199608220854.JAA28160@paperboy.ccc.nottingham.ac.uk>, you write: >From: Barry Tesman >Date: Fri, 26 Jul 1996 09:37:43 -0400 (EDT) >Subject: Matrices > >I am trying to produce the following "labelled" matrix with latex: > > B1 B2 B3 > / \ > a1 | 8 10 4 | >A = a2 | 11 5 18 | > a3 | 27 6 18 | > \ / > >I tried to use array/tabbing/tabular but am having trouble placing >the left and right large parentheses around the matrix but not the labels. >Any ideas would be most helpful. _______________________________________________________________________________ Dennis Kletzing Department of Mathematics & Computer Science Stetson University DeLand, Florida kletzing@totcon.com ------------------------------ From: "K. Berry" Date: Fri, 23 Aug 1996 15:56:09 -0400 Subject: Re: TeXhax Digest V96 #12 In the times font ptmr8r, the usual positions for the left single quote "60 and for the right single quote "27 contain instead of the symbols found in the font cmr a symbol that looks like a grave accent and a symbol that looks like a vertical prime, respectively. How is one to get matched single quotes? There is an older font rptmr that has the usual single quotes in those positions. Is there some way to use the font ptmr8r and still get the usual single quotes, for example, by means of some trick using virtual fonts and rptmr? Use ptmr (close to plain-compatible) or ptmr8q (Cork), which have the usual quotes in the usual positions. Those are virtual fonts based on 8r. I wouldn't call them tricks, exactly. ptmr8r is close to Windows-compatible, for reasons explained in the 8r.enc source file. Its primary purpose is to make all characters commonly present in Type 1 fonts available for typesetting (i.e., with virtual fonts), not to be the One True Encoding for typesetting itself. If you want to use 8r itself, then you'll have to make ` and ' active and have them typeset the right character. You probably don't want to do this. ------------------------------ From: Erik Frambach Date: Thu, 29 Aug 1996 10:33:26 GMT+0200 Subject: (Fwd) Repeated announcement T&G course NTG - ------- Forwarded Message Follows ------- Date: Mon, 26 Aug 1996 13:03:16 +0200 Reply-to: Netherlands Tex users-Group , Kees van der Laan From: Kees van der Laan Subject: Repeated announcement T&G course NTG ---------------------------------------------------------- |Announcement NTG's one-day low-budget no-nonsense course| | | | (La)TeX and Graphics | ---------------------------------------------------------- What? MetaPost, PostScript, mftoeps... (no font design) Language? English Teacher? Boguslaw Jackowksi Where? University of Utrecht When? The day before *or* after NTG's fall meeting of 24 Oct. (To be announced late August) Costs? Fl 50 members of NTG or other LUGs; non-members Fl 150 Subsription? Treasurer NTG, Giro 1306238, Eindhoven (Do mention TeX and Graphics course) Information? Kees van der Laan, cgl@rc.service.rug.nl Literature? Jackowksi's METAFONT booklet will be in English available. Have a look at Hobby's CSTR 162, A user manual for MetaPost, from netlib@research.att.com with message send 162 from research/cstr or copy it from NTG's 4AllTeX CD-ROM When the number of subscriptions is insufficient of 7 September---ultimate date for registration--- the course will be cancelled (and money refunded). - ---Kees--- ------------------------------ From: Mark Freeman Date: Thu, 29 Aug 1996 10:00:09 GMT Subject: Memory question Sorry if this is a FAQ or a newbie question, but I am in need of some help. I have tried crawling the web pages but with little success... I am running LaTeX2.09 (dated 9/1/90) under DOS. I am towards the end of writing my thesis and am currently trying to compile it as one document (rather than individual chapters) using \include instructions. On first compilation it runs fine, and will continue to compile on each subsequent compilation UNLESS I have run BibTeX first. In this case, I can only get it to recompile if I instruct "del *.aux" prior to recompilation - obviously not very satisfactory. The number of different references is around 180 at present (although there are many more \citeasnoun instructions). The error message I am getting on recompiling after running BibTeX is insufficient memory. On looking at the .log file it appears to be "save size" that is crashing. After first compilation the memory counter is "362s" and then on second compilation I get "601s ... out of ... 600s". Other sections of memory appear to be fine. I have tried to reduce the number of \citeasnoun, but this appears to make no difference. If I edit my .aux file to manually remove about 50 of the 180 citations it will recompile with no problem. Any ideas about what I should do to get it to recompile with no crashes after running BibTeX? I would be very grateful if you would email me direct with any potential solutions. Thanking you in advance for your help. Mark Freeman. ------------------------------ From: Clinton Arokianathan Date: Fri, 30 Aug 1996 15:02:23 +0100 Subject: ispell problems I have recently downloaded ispell version 3.1. Unfortunately it is still the US dictionary despite using the Makefile in /languages/british. We also have the /langages/english folder. What lables need to be changed in local.h? I would be grateful if you can help Clint - --------------------------------------------------------------------- Clinton R Arokianathan Nanoelectronics Research Centre Department of Electronics and Electrical Engineering University of Glasgow Glasgow G12 8QQ Scotland phone: +44-(0)141-339-8855 Ext 6024 (office) 8349 (laboratory) fax: +44-(0)141-330-4907 e-mail: clint@elec.gla.ac.uk - --------------------------------------------------------------------- ------------------------------ From: cross@seraph1.sewanee.edu (Clay C. Ross) Date: Fri, 11 Oct 1996 09:30:23 +0100 Subject: Re: word to latex converter At 12:24 PM 10/10/96, Kevin R. Vixie wrote: * I am interested in your web site dealing with word to tex conv. but it * seems to be offline right now. * * Kevin R. Vixie This was my reply: I do not now have, nor ever have had such a site. I HAVE answered *many* questions since spring about rtf2latex and latex2rtf, two programs that others wrote. To find the flavor you need, look them up using a web browser. Also, rtf2tex and tex2rtf probably exist. Find them by searching. - -- I add this for this transmission: -- There seems to be a great deal of interest in converters to- and from- Word (on many platforms). All I know about them is that the conversions work as follows: Word Doc. -> Word(Save As) -> RTF(text)Doc.-> rtf2latex -> LaTeX Doc. -> LaTeX or Latex Document -> latex2rtf -> Word(read and convert RTF) -> Word Document Some TeX/LaTeX expert could provide a valuable service by setting up a site dedicated to such converters. I am almost totally ignorant of the issues; for me to become involved would be foolhardy. Please, would an informed person step up to provide this service (a Converter Site)? CCR +++++++++++++++++++++++++++++++ Clay C. Ross (615) 598-1301 http://cross.sewanee.edu/ Mathematics & Computer Science The University of the South Sewanee, Tennessee 37383-1000 ------------------------------ From: l44@iwd.uni-bremen.de (Roland Weibezahn) Date: Tue, 15 Oct 1996 09:23:56 +0200 Subject: Announce: xtem X11-TeX-Menu 5.23 Dear xtem users, We have finished the new version (xtem_TeXMenu.5.23) of "xtem", an X11-TeX-menu which runs with the new Tcl/Tk/TclX (Tcl7.5/ and Tcl7.6-beta1) as well as with the old version (Tcl7.4). In addition to adapting xtem to the new Tcl/Tk, we have done errror corrections and added new features, such as: - installation (and maintenance of the logfiles) is much easier with the (new) installation procedure, - (optional) logfile analyze after TeX run, you can then click at each error message (incl. overfull/underfull boxes) in order to edit the texfile at the corresponding position, - "quick&dirty" button for quick TeXing and previewing of a small text fragment (including preamble), - the edit file can be selected from other directories than the main file, - (optional) vertical scrollbars for the text widgets, - revision of all bindings (conforming behaviour: actions are done at button release), - LaTeX syntax got it's own button now. Most setting files from older versions of xtem may be kept unchanged: - mkcommand.vst must be updated - texfmt.vst should be updated to enable "quick&dirty" - logform.vst one line concerning logfile analyze can/should be added You will find all files on our file server: http://ftp.iwd.uni-bremen.de/xtem/xtem_texmenu.html or ftp://ftp.iwd.uni-bremen.de/pub/tex/xtem.v5/xtem_texmenu.5.23.tar.gz ftp://ftp.iwd.uni-bremen.de/pub/tex/xtem.v5/xtem_texmenu_5eng.ps.gz ftp://ftp.iwd.uni-bremen.de/pub/tex/xtem.v5/xtem_texmenu_5ger.ps.gz Here you may also find the sources: Tcl7.5, Tk4.1, TclX7.5.2 ftp://ftp.iwd.uni-bremen.de/pub/tcl/tcl7.4p2.tar.gz ftp://ftp.iwd.uni-bremen.de/pub/tcl/tk4.0p2.tar.g ftp://ftp.iwd.uni-bremen.de/pub/tcl/tclX7.4a-p1.tar.gz We have uploaded the new xtem version to the neosoft server into "/pub/tcl/incoming/", and it's already mirrored to the CTAN servers, so you also may get all the material from ftp://ftp.neosoft.com/pub/tcl/NEW/xtem_* and ftp://ftp.dante.de/tex-archive/support/xtem_texmenu/xtem.v5/* and the other CTAN servers. For those who don't know xtem_TeXMenu up to now, we give a short overview: xtem provides for a simple and comfortable graphical user interface to control the following facilities: - file and directory selection, directory creation, - editor (vi, emacs, ...) including additional windows for the LaTeX-syntax (using hypertext) and examples, - TeX, LaTeX, ..., - previewer (ghostview, xdvi, TkDvi, ...), - printing (including comfortable printer selection, ...), - syntax and spelling check, - makeindex, - bibtex, - additional programs as required, . . . Online help is available for all the buttons and windows by simple mouse click. xtem is written for Unix platforms and has been tested on many systems. Thanks, Roland Weibezahn - --- Dr. Roland Weibezahn weibezahn@iwd.uni-bremen.de phone: +49-421-218-3532 University Bremen, IWD, postbox: 330440, 28334 Bremen, Germany http://ftp.iwd.uni-bremen.de/xtem/xtem_texmenu.html (the xtem_TeXMenu project) ------------------------------ End of TeXhax Digest V96 #13 **************************** About TeXhax... Please send contributions to: TeXhax@tex.ac.uk Subscription and unsubscription requests: send a one line mail message to TeXhax-Request@tex.ac.uk containing only the line subscribe texhax or unsubscribe texhax If you have problems with un/subscribing, please mail texhax-owner@nottingham.ac.uk For information on the TeX Users Group, please send a message to TUG@TUG.org, or write TeX Users Group, 1850 Union Street, #1637 San Francisco CA 94123 (phone: 1 415 982 8449, fax: 1 415 982 8559) Backnumbers of all the digests are stored in the Comprehensive TeX Archive Network (CTAN) and can be retrieved on the Internet by anonymous ftp. The hosts comprising CTAN include, among others, ftp.dante.de (129.69.1.12) -- Germany ftp.tex.ac.uk (128.232.1.87) -- UK Please use your nearest server, to keep network load down. The file /tex-archive/CTAN.sites on each of these hosts gives a list of other sites which maintain full or partial mirrors of the CTAN. Alternatively, finger ctan_us@ftp.shsu.edu for full details. TeXhax Digest back issues are filed below /tex-archive/digests/texhax/ Keyword-In-Context indexes are filed in /tex-archive/digests/indexes/ A Hypermail version of TeXhax is also available on the World-Wide Web at URL http://www.tex.ac.uk/tex-archive/digests/hyper/ \bye From texhax-owner@nottingham.ac.uk Thu Dec 19 12:35:05 1996 Flags: 000000000001 Received: from nottingham.ac.uk (jess.ccc.nottingham.ac.uk [128.243.40.193]) by csc-sun.math.utah.edu (8.7.6/8.7.3) with SMTP id MAA24758 for ; Thu, 19 Dec 1996 12:34:55 -0700 (MST) Received: from nottingham.ac.uk [128.243.40.194] by nottingham.ac.uk with smtp (Exim 1.58 #10) id 0van3G-0004Kj-00; Thu, 19 Dec 1996 18:18:38 +0000 Received: from dryden.ccc.nottingham.ac.uk [128.243.40.195] by nottingham.ac.uk with smtp (Exim 1.58 #10) id 0van3F-0001e9-00; Thu, 19 Dec 1996 18:18:37 +0000 Received: from unix.ccc.nottingham.ac.uk [128.243.40.215] by dryden.ccc.nottingham.ac.uk with smtp (Exim 1.58 #10) id 0van3E-00041J-00; Thu, 19 Dec 1996 18:18:36 +0000 Received: from unix.ccc.nottingham.ac.uk [127.0.0.1] by unix.ccc.nottingham.ac.uk with esmtp (Exim 1.58 #2) id 0van3D-00070i-00; Thu, 19 Dec 1996 18:18:35 +0000 Prev-Resent: Thu, 19 Dec 1996 18:16:56 +0000 Prev-Resent: texhax@nottingham.ac.uk Message-Id: From: cczdao@unix.ccc.nottingham.ac.uk Date: Thu, 19 Dec 1996 18:18:35 +0000 >From texhax-digest-owner@nottingham.ac.uk Thu Dec 19 18: 12:44 1996 Received: from nottingham.ac.uk (jess.ccc.nottingham.ac.uk [128.243.40.193]) by granby.ccc.nottingham.ac.uk (8.6.12/8.6.12) with SMTP id SAA20933 for ; Thu, 19 Dec 1996 18:12:43 GMT Received: from majordom by nottingham.ac.uk with local (Exim 1.58 #10) id 0vamxI-0004Im-00; Thu, 19 Dec 1996 18:12:28 +0000 From: owner-texhax-digest@nottingham.ac.uk To: texhax-digest@nottingham.ac.uk Subject: TeXhax Digest V96 #15 Reply-To: TeXhax@tex.ac.uk Errors-To: owner-texhax-digest Precedence: bulk Message-Id: Date: Thu, 19 Dec 1996 18:12:28 +0000 Resent-To: texhax-outgoing@nottingham.ac.uk Resent-Date: Thu, 19 Dec 1996 18:18:34 +0000 Resent-Message-ID: <26951.851019514@unix.ccc.nottingham.ac.uk> Resent-From: David Osborne TeXhax Digest Thursday, 19 December 1996 Volume 96 : Number 015 (incorporating UKTeX Digest) Today's Topics: New version of AMS-LaTeX released Latex e-TeX is released Plain TeX query about \font correct way to write Makefile? ---------------------------------------------------------------------- From: bbeeton Date: Fri, 08 Nov 1996 08:24:45 -0500 (EST) Subject: New version of AMS-LaTeX released An interim release of the AMS-LaTeX version 1.2 macro collection has been posted. This release fixes a number of bugs, and documents others that have not yet been fixed, but are known and are on the list for attention when time becomes available. The documentation has been extensively revised and users are requested to pay careful attention to the READ.ME file. In particular, AMS-LaTeX 1.2 now requires a version of LaTeX2e no earlier than December 1994. The canonical home site for this collection is e-math.ams.org for ftp, or at http://www.ams.org/tex for those who prefer a Web connection. The collection has also been mirrored onto CTAN (locations shown are relative to the root of the tex archive at the particular site): location on e-math: location on CTAN: /pub/tex/amslatex under macros/latex/packages/amslatex The other AMS collections are available from the same AMS servers and have now been installed on CTAN in these new locations: location on e-math: location on CTAN: /pub/tex/amsfonts under fonts/amsfonts /pub/tex/amsltx11 under macros/latex209/contrib/amslatex /pub/tex/amstex under macros/amstex At CTAN, the changes have been made at ftp.tex.ac.uk and ftp.dante.de; they will propagate to the mirrors in due course. Only the AMS-LaTeX collection has been updated; some minor changes are under construction in the AMSFonts and AMS-TeX collections, and a separate announcement will be made when these are posted. ------------------------------ From: zadokh@rafael.co.il (Zadok Hougui) Date: Mon, 11 Nov 1996 22:23:22 +0200 Subject: Latex Hi Do you know where I could download a copy of Latex for use on an IBM PC compatible? (for a windows 3.1 or windows 95 environment) Thanks Zadok ========================================================================== Sadok Hougui 32 Lehi St. Kiryat Bialik, 27000 Israel Home phone: (972)-4-8740167 e-mail: zadokh@rafael.co.il ------------------------------ From: Philip Taylor Date: Wed, 13 Nov 1996 17:55:17 GMT Subject: e-TeX is released Dear Colleagues -- I am delighted to be able to tell you that after a protracted period of design and development, e-TeX is now formally released. In summary, e-TeX is an evolutionary (rather that revolutionary) development of TeX, and is 100%-compatible, including compatibility at the level of the TRIP test. However, it is also capable of operating in "extended" mode, in which case it adds approximately 30 new primitives to the TeX language whilst remaining completely compatible with TeX if none of the new primitives are used, and in "enhanced" mode, in which case it adds functionality (for the first release, TeX--XeT) at the expense of compatibility. Reference implementations are available for VMS (both VAX & AXP) and MS/DOS, the former implementation being primarily the work of Christian Spieler whilst the latter is the work of Peter Breitenlohner. As this message is being sent to TeX-Implementors as well as to the NTS and TeX lists, we hope that new implementations will become available in the near future. For further information on e-TeX, and for access to the sources and reference implementations, please see: http://www.rhbnc.ac.uk/e-TeX/ Philip Taylor, for and on behalf of the e-TeX team / NTS group. ------------------------------ From: cgm@ssci.liv.ac.uk (Colin Mason) Date: Wed, 27 Nov 1996 14:45:13 GMT Subject: Plain TeX query about \font If you do: \font\titlefont=cmssdc10 \ifx\titlefont\nullfont \wlog{Using \string\nullfont}% \fi \bye You get the following message in the log file: Using \nullfont But if there is a blank line between the \font... and the \ifx... this does not happen. I would not expect to get the message anyway so why does it happen and why does inserting a blank line cure it? Thanks in advance for any help. Colin Mason. ------------------------------ From: Steve Kelem Date: Fri, 06 Dec 1996 09:30:11 -0800 Subject: correct way to write Makefile? What's the correct way to create a Makefile for LaTeX? I'm not sure how to capture the business about having to run: latex (Extract citations.) bibtex (Create the bibliography.) latex (Incorporate bibliography.) latex (Resolve references.) Thanks, /7\'7 Steve Kelem (408)879-5347 Steve.Kelem@xilinx.com \\ ` Xilinx FAX: (408)377-3259 // 2100 Logic Drive \\/.\ San Jose, California 95124 So far, I have the following, but it doesn't capture the above requirements. .SUFFIXES : $(LATEX_SUFFIXES) $(SUFFIXES) LATEX_SUFFIXES = .aux .bbl .blg .dvi .ilg .idx .ind .lof .log .lot .toc all : pop.ps %.ps : %.dvi dvips OPTIONS $*.dvi %.ind : %.idx makeindex $* %.bbl : %.aux bibtex $* %.dvi %.aux : %.tex latex $*.tex ------------------------------ End of TeXhax Digest V96 #15 **************************** About TeXhax... Please send contributions to: TeXhax@tex.ac.uk Subscription and unsubscription requests: send a one line mail message to TeXhax-Request@tex.ac.uk containing only the line subscribe texhax or unsubscribe texhax If you have problems with un/subscribing, please mail texhax-owner@nottingham.ac.uk For information on the TeX Users Group, please send a message to TUG@TUG.org, or write TeX Users Group, 1850 Union Street, #1637 San Francisco CA 94123 (phone: 1 415 982 8449, fax: 1 415 982 8559) Backnumbers of all the digests are stored in the Comprehensive TeX Archive Network (CTAN) and can be retrieved on the Internet by anonymous ftp. The hosts comprising CTAN include, among others, ftp.dante.de (129.69.1.12) -- Germany ftp.tex.ac.uk (128.232.1.87) -- UK Please use your nearest server, to keep network load down. The file /tex-archive/CTAN.sites on each of these hosts gives a list of other sites which maintain full or partial mirrors of the CTAN. Alternatively, finger ctan_us@ftp.shsu.edu for full details. TeXhax Digest back issues are filed below /tex-archive/digests/texhax/ Keyword-In-Context indexes are filed in /tex-archive/digests/indexes/ A Hypermail version of TeXhax is also available on the World-Wide Web at URL http://www.tex.ac.uk/tex-archive/digests/hyper/ \bye From beebe@math.utah.edu Mon Mar 3 09:29:50 1997 Flags: 000000000000 Received: from plot79.math.utah.edu (beebe@plot79.math.utah.edu [128.110.198.3]) by csc-sun.math.utah.edu (8.8.4/8.8.4) with ESMTP id JAA25572 for ; Mon, 3 Mar 1997 09:29:50 -0700 (MST) From: "Nelson H. F. Beebe" Received: (from beebe@localhost) by plot79.math.utah.edu (8.8.4/8.8.4) id JAA09371 for beebe; Mon, 3 Mar 1997 09:29:48 -0700 (MST) Date: Mon, 3 Mar 1997 09:29:48 -0700 (MST) Message-Id: <199703031629.JAA09371@plot79.math.utah.edu> To: beebe@math.utah.edu Subject: TeXhax Digest V96 #14 (Message texhax/v96:14) From: owner-texhax-digest To: texhax-digest Subject: TeXhax Digest V96 #14 Reply-To: TeXhax@tex.ac.uk Errors-To: owner-texhax-digest Precedence: bulk TeXhax Digest Friday, 1 November 1996 Volume 96 : Number 014 (incorporating UKTeX Digest) Today's Topics: unsuscribe Phone: +44 171 380 7293 Converting LaTex2e to HTML Re: TeXhax Digest V96 #13 Downloading TeX & LaTeX Plain TeX query about \csname ... \endcsname Memory question 'none' ---------------------------------------------------------------------- From: Dave Elliman Date: Tue, 15 Oct 96 12:07:16 +0100 Subject: unsuscribe ------------------------------ From: Russel Winder Date: Tue, 15 Oct 1996 14:11:08 +0100 Subject: Phone: +44 171 380 7293 Jose, > From: Jose Manuel Souto Menendez > Date: Fri, 4 Oct 1996 19:14:51 +0100 (WET DST) > Subject: Answers > > I am writing a book with Latex and I would like to know how it is posible > to write the answers of some of the exercises in such a way that that > they appear at the end of the book. Are there any macros to solve > this problem? (I am using AMSLaTeX, and with LaTeX2e). > Thank you very much, > Jose M. Souto I do not know of any macros for doing this, I have to admit I didn't look that hard. I wrote my own macros for doing this, initially as TeX macros then as LaTeX(209|2e) environment(s). My basic algorithm was to write the exercise and answers out to a file and then read them in at the appropriate place in the text. This is not entirely trivial as the writing has to be of the original code; the verbatim stuff from the standard TeX/LaTeX(209|2e) came in very handy at this point. I suppose I should have turned these macros into a separate package and submitted it, if there is enough interest I can do this. Otherwise I could let you have a copy of my macros. Russel. ======================================================================= Dr Russel Winder Reader in Software Engineering Editor-in-Chief, Object Oriented Systems Information Systems Research Group Department of Computer Science Phone: +44 (0)171 380 7293 University College London Fax: +44 (0)171 387 1397 Gower Street EMail: R.Winder@cs.ucl.ac.uk London WC1E 6BT UK URL: http://www.cs.ucl.ac.uk/staff/R.Winder/ ======================================================================= ------------------------------ From: reynoldd@ccmail.dcu.ie Date: Wed, 16 Oct 1996 08:53:11 +0000 (GMT) Subject: Converting LaTex2e to HTML Dear TeXhax Digest, Presumably there's alot of work being done on how to convert LaTex2e to HTML. I want to put lecture notes with equations onto our intranet. Can you point me to where I might find relevant advice. Latex2html seems a good answer for unix users. However I use DOS, WIN 3.11 and OS2 Warp. I did find correspondence on a newsgroup about compiling a DOS version of LaTex2HTML, but there was much discussion about perl and fork, about which I am ignorant. Any advice would be gratefully received. Regards, David Reynolds ------------------------------ From: Andreas Schlechte Date: Thu, 17 Oct 1996 18:48:23 +0100 (MET) Subject: Re: TeXhax Digest V96 #13 Hi, In reply to > From: Jose Manuel Souto Menendez > Date: Fri, 4 Oct 1996 19:14:51 +0100 (WET DST) > Subject: Answers > I am writing a book with Latex and I would like to know how it is posible > to write the answers of some of the exercises in such a way that that > they appear at the end of the book. Are there any macros to solve > this problem? (I am using AMSLaTeX, and with LaTeX2e). > Thank you very much, > Jose M. Souto Well, you can either use the macros, Knuth is using in his TeXbook. But it's (a bit) difficult to implement these. I use some other macros, based on the package "moreverb.sty", which must be included. ---- solution.sty ---- % % open new file solution.tex for writing % \newwrite\solutionout \immediate\openout\solutionout=solution.tex % % define environment solutionwrite % \def\solutionwrite{% \@bsphack \let\do\@makeother\dospecials \catcode`\^^M\active \catcode`\^^I=12 \def\verbatim@processline{\immediate\write\solutionout{\the\verbatim@line}}% \verbatim@start} \def\endsolutionwrite{\@esphack} % % Use this command to close file % \def\closesolutionfile{\immediate\closeout\solutionout} ---- end ---- With these package you will be able to write any string into the file. Here is an example: \documentclass{book} \usepackage{moreverb,solution} \begin{document} \begin{solutionwrite} You may use any character in this environment. its although possible to write TeX-command into the file: \begin{quote} Like this quoted block. \end{quote} \end{solutionwrite} %... \closesolutionfile \input solution.tex \end{document} The only restriction is, that you might not write the string \end{solutionwrite} into the file. In my opinion there is rather no need for this. Hope, this helps a little bit at least, Andi - -- >>>>>>>>>>>>>>>>>>>>>>>>>>>>>>>>>>>>><<<<<<<<<<<<<<<<<<<<<<<<<<<<<<<<<<<<<< > Andreas Schlechte Tel.: +49-5323-1289 Q < > Am Klepperberg 2 +49-5323-9499-7 < > 38678 Clausthal-Zellerfeld Fax.: +49-5323-9499-3 < > Andreas.Schlechte@tu-clausthal.de http://www.tu-clausthal.de/~inas < >>>>>>>>>>>>>>>>>>>>>>>>>>>>>>>>>>>>><<<<<<<<<<<<<<<<<<<<<<<<<<<<<<<<<<<<<< ------------------------------ From: "P. WISUTHSERIWONG" Date: Fri, 18 Oct 1996 15:07:26 +0000 Subject: Downloading TeX & LaTeX Hi there, I'm wondering if you could help. I'm interested in using LaTeX but have no idea where & how I can get the programmes. Could you tell me the actual file names I need to download from the ctan site please? Oh, do I have to have TeX in order to run LaTeX? Thanks very much for your time. P. Wisuthseriwong ------------------------------ From: cgm@ssci.liv.ac.uk (Colin Mason) Date: Tue, 22 Oct 1996 15:08:54 GMT Subject: Plain TeX query about \csname ... \endcsname If you do: \tracingrestores=1 {\expandafter\xdef\csname 777\endcsname{555}} \bye you get the message: {retaining \777=macro:->555} in the log file. Page 301 of the TeXbook says that nothing needs to go on the save stack at the time of a global assignment so I was initially surprised to get the message. However, page 213 says that \csname ... \endcsname defines the control sequence to be like \relax if it is not currently defined. So, it would appear that \csname ... \endcsname first locally defines \777 to be like \relax and the \xdef then globally redefines it to be 555. If you then do: \tracingrestores=1 {\globaldefs=1 \expandafter\xdef\csname 777\endcsname{555}} \bye it makes no difference. Why? Shouldn't the \csname ... \endcsname globally define \777 to be like \relax? Or is this some strange exception to the rules? Thanks in advance for any help. Colin Mason. ------------------------------ From: Mark Freeman Date: Mon, 28 Oct 1996 14:29:49 GMT Subject: Memory question On 29/8/96 I posted a question to TeXhax on a memory problem that I was encountering running LaTeX2.09 under Dos on a document with many references. The following reply was sent to me by Jonathan Fine: > So far as I can tell, the problem is with the lines of code > > \def\bibcite#1#2{\global\@namedef{b@#1}{#2}} > > \def\@namedef#1{\expandafter\def\csname #1\endcsname} > in the main source file for latex, version 2.09. (Comments on LaTeX > 2e appear later.) > > Please place the definition > > \def\bibcite#1#2{\begingroup\expandafter\endgroup\expandafter > > \gdef\csname b@#1\endcsname{#2}} > somewhere in the preamble to your document. If I am correct, then > the problem will go away. > > Explanation. When \csname ... \endcsname produces an undefined > control sequence, it is as a local assignment given the value \relax. > This has the side effect of occupying space in the save stack. The > replacement definition does not remove this side effect, but makes it > harmless. The \expandafter's function so that the local definition > is valid only within the > > \begingroup\expandafter\endgroup > group of \bibcite. At the end of this group, the previous value of > \undefined is restored for \b@#1, and this frees up the space in the > save stack. Use \tracingall to get the gory details. > > For another explanation, see my article in Baskerville (4)1, > published 1994. In my article I wrote "This [side effect] can cause > problems in processing LaTeX documents which have a lot of > cross-references." So far as I can tell, this problem remains with > LaTeX 2e. As I am running \harvardcite as opposed to \bibcite I made the following minor adjustment to Jonathan's proposed solution: \def\harvardcite#1#2#3#4{\begingroup\expandafter\endgroup \expandafter\gdef\csname b@#1\endcsname{#2} \begingroup\expandafter\endgroup \expandafter\gdef\csname bhf@#1\endcsname{#2} \begingroup\expandafter\endgroup \expandafter\gdef\csname bha@#1\endcsname{#3} \begingroup\expandafter\endgroup \expandafter\gdef\csname bhy@#1\endcsname{#4}} This appears to have fixed my problem. I hope that others might find this correspondance useful. Regards, Mark Freeman. ------------------------------ From: "Robert Greer c/o CUNY 212-346-8450 (fax 346-8453)" Date: Fri, 01 Nov 96 10:43:13 EST Subject: 'none' Acknowledge-To: ------------------------------ End of TeXhax Digest V96 #14 **************************** About TeXhax... Please send contributions to: TeXhax@tex.ac.uk Subscription and unsubscription requests: send a one line mail message to TeXhax-Request@tex.ac.uk containing only the line subscribe texhax or unsubscribe texhax If you have problems with un/subscribing, please mail texhax-owner@nottingham.ac.uk For information on the TeX Users Group, please send a message to TUG@TUG.org, or write TeX Users Group, 1850 Union Street, #1637 San Francisco CA 94123 (phone: 1 415 982 8449, fax: 1 415 982 8559) Backnumbers of all the digests are stored in the Comprehensive TeX Archive Network (CTAN) and can be retrieved on the Internet by anonymous ftp. The hosts comprising CTAN include, among others, ftp.dante.de (129.69.1.12) -- Germany ftp.tex.ac.uk (128.232.1.87) -- UK Please use your nearest server, to keep network load down. The file /tex-archive/CTAN.sites on each of these hosts gives a list of other sites which maintain full or partial mirrors of the CTAN. Alternatively, finger ctan_us@ftp.shsu.edu for full details. TeXhax Digest back issues are filed below /tex-archive/digests/texhax/ Keyword-In-Context indexes are filed in /tex-archive/digests/indexes/ A Hypermail version of TeXhax is also available on the World-Wide Web at URL http://www.tex.ac.uk/tex-archive/digests/hyper/ \bye From beebe@math.utah.edu Mon Mar 3 09:30:46 1997 Flags: 000000000000 Received: from plot79.math.utah.edu (beebe@plot79.math.utah.edu [128.110.198.3]) by csc-sun.math.utah.edu (8.8.4/8.8.4) with ESMTP id JAA25603 for ; Mon, 3 Mar 1997 09:30:45 -0700 (MST) From: "Nelson H. F. Beebe" Received: (from beebe@localhost) by plot79.math.utah.edu (8.8.4/8.8.4) id JAA09376 for beebe; Mon, 3 Mar 1997 09:30:44 -0700 (MST) Date: Mon, 3 Mar 1997 09:30:44 -0700 (MST) Message-Id: <199703031630.JAA09376@plot79.math.utah.edu> To: beebe@math.utah.edu Subject: TeXhax Digest V96 #15 (Message texhax/v96:15) Resent: Thu, 19 Dec 1996 18:18:37 +0000 Resent: texhax-outgoing@nottingham.ac.uk Resent: Thu, 19 Dec 1996 18:16:56 +0000 Resent: texhax@nottingham.ac.uk >From texhax-digest-owner@nottingham.ac.uk Thu Dec 19 18:12:44 1996 Received: from nottingham.ac.uk (jess.ccc.nottingham.ac.uk [128.243.40.193]) by granby.ccc.nottingham.ac.uk (8.6.12/8.6.12) with SMTP id SAA20933 for ; Thu, 19 Dec 1996 18:12:43 GMT Received: from majordom by nottingham.ac.uk with local (Exim 1.58 #10) id 0vamxI-0004Im-00; Thu, 19 Dec 1996 18:12:28 +0000 From: owner-texhax-digest@nottingham.ac.uk To: texhax-digest@nottingham.ac.uk Subject: TeXhax Digest V96 #15 Reply-To: TeXhax@tex.ac.uk Errors-To: owner-texhax-digest Precedence: bulk Message-Id: Date: Thu, 19 Dec 1996 18:12:28 +0000 TeXhax Digest Thursday, 19 December 1996 Volume 96 : Number 015 (incorporating UKTeX Digest) Today's Topics: New version of AMS-LaTeX released Latex e-TeX is released Plain TeX query about \font correct way to write Makefile? ---------------------------------------------------------------------- From: bbeeton Date: Fri, 08 Nov 1996 08:24:45 -0500 (EST) Subject: New version of AMS-LaTeX released An interim release of the AMS-LaTeX version 1.2 macro collection has been posted. This release fixes a number of bugs, and documents others that have not yet been fixed, but are known and are on the list for attention when time becomes available. The documentation has been extensively revised and users are requested to pay careful attention to the READ.ME file. In particular, AMS-LaTeX 1.2 now requires a version of LaTeX2e no earlier than December 1994. The canonical home site for this collection is e-math.ams.org for ftp, or at http://www.ams.org/tex for those who prefer a Web connection. The collection has also been mirrored onto CTAN (locations shown are relative to the root of the tex archive at the particular site): location on e-math: location on CTAN: /pub/tex/amslatex under macros/latex/packages/amslatex The other AMS collections are available from the same AMS servers and have now been installed on CTAN in these new locations: location on e-math: location on CTAN: /pub/tex/amsfonts under fonts/amsfonts /pub/tex/amsltx11 under macros/latex209/contrib/amslatex /pub/tex/amstex under macros/amstex At CTAN, the changes have been made at ftp.tex.ac.uk and ftp.dante.de; they will propagate to the mirrors in due course. Only the AMS-LaTeX collection has been updated; some minor changes are under construction in the AMSFonts and AMS-TeX collections, and a separate announcement will be made when these are posted. ------------------------------ From: zadokh@rafael.co.il (Zadok Hougui) Date: Mon, 11 Nov 1996 22:23:22 +0200 Subject: Latex Hi Do you know where I could download a copy of Latex for use on an IBM PC compatible? (for a windows 3.1 or windows 95 environment) Thanks Zadok ========================================================================== Sadok Hougui 32 Lehi St. Kiryat Bialik, 27000 Israel Home phone: (972)-4-8740167 e-mail: zadokh@rafael.co.il ------------------------------ From: Philip Taylor Date: Wed, 13 Nov 1996 17:55:17 GMT Subject: e-TeX is released Dear Colleagues -- I am delighted to be able to tell you that after a protracted period of design and development, e-TeX is now formally released. In summary, e-TeX is an evolutionary (rather that revolutionary) development of TeX, and is 100%-compatible, including compatibility at the level of the TRIP test. However, it is also capable of operating in "extended" mode, in which case it adds approximately 30 new primitives to the TeX language whilst remaining completely compatible with TeX if none of the new primitives are used, and in "enhanced" mode, in which case it adds functionality (for the first release, TeX--XeT) at the expense of compatibility. Reference implementations are available for VMS (both VAX & AXP) and MS/DOS, the former implementation being primarily the work of Christian Spieler whilst the latter is the work of Peter Breitenlohner. As this message is being sent to TeX-Implementors as well as to the NTS and TeX lists, we hope that new implementations will become available in the near future. For further information on e-TeX, and for access to the sources and reference implementations, please see: http://www.rhbnc.ac.uk/e-TeX/ Philip Taylor, for and on behalf of the e-TeX team / NTS group. ------------------------------ From: cgm@ssci.liv.ac.uk (Colin Mason) Date: Wed, 27 Nov 1996 14:45:13 GMT Subject: Plain TeX query about \font If you do: \font\titlefont=cmssdc10 \ifx\titlefont\nullfont \wlog{Using \string\nullfont}% \fi \bye You get the following message in the log file: Using \nullfont But if there is a blank line between the \font... and the \ifx... this does not happen. I would not expect to get the message anyway so why does it happen and why does inserting a blank line cure it? Thanks in advance for any help. Colin Mason. ------------------------------ From: Steve Kelem Date: Fri, 06 Dec 1996 09:30:11 -0800 Subject: correct way to write Makefile? What's the correct way to create a Makefile for LaTeX? I'm not sure how to capture the business about having to run: latex (Extract citations.) bibtex (Create the bibliography.) latex (Incorporate bibliography.) latex (Resolve references.) Thanks, /7\'7 Steve Kelem (408)879-5347 Steve.Kelem@xilinx.com \\ ` Xilinx FAX: (408)377-3259 // 2100 Logic Drive \\/.\ San Jose, California 95124 So far, I have the following, but it doesn't capture the above requirements. .SUFFIXES : $(LATEX_SUFFIXES) $(SUFFIXES) LATEX_SUFFIXES = .aux .bbl .blg .dvi .ilg .idx .ind .lof .log .lot .toc all : pop.ps %.ps : %.dvi dvips OPTIONS $*.dvi %.ind : %.idx makeindex $* %.bbl : %.aux bibtex $* %.dvi %.aux : %.tex latex $*.tex ------------------------------ End of TeXhax Digest V96 #15 **************************** About TeXhax... Please send contributions to: TeXhax@tex.ac.uk Subscription and unsubscription requests: send a one line mail message to TeXhax-Request@tex.ac.uk containing only the line subscribe texhax or unsubscribe texhax If you have problems with un/subscribing, please mail texhax-owner@nottingham.ac.uk For information on the TeX Users Group, please send a message to TUG@TUG.org, or write TeX Users Group, 1850 Union Street, #1637 San Francisco CA 94123 (phone: 1 415 982 8449, fax: 1 415 982 8559) Backnumbers of all the digests are stored in the Comprehensive TeX Archive Network (CTAN) and can be retrieved on the Internet by anonymous ftp. The hosts comprising CTAN include, among others, ftp.dante.de (129.69.1.12) -- Germany ftp.tex.ac.uk (128.232.1.87) -- UK Please use your nearest server, to keep network load down. The file /tex-archive/CTAN.sites on each of these hosts gives a list of other sites which maintain full or partial mirrors of the CTAN. Alternatively, finger ctan_us@ftp.shsu.edu for full details. TeXhax Digest back issues are filed below /tex-archive/digests/texhax/ Keyword-In-Context indexes are filed in /tex-archive/digests/indexes/ A Hypermail version of TeXhax is also available on the World-Wide Web at URL http://www.tex.ac.uk/tex-archive/digests/hyper/ \bye